Vous êtes sur la page 1sur 37

Sinan Salim

A well-defined radiolucency at the apex of a The best method to prevent root canal obstruction
nonvital tooth is most likely a/an during the instrumentation phase of endodontic
A. lateral periodontal cyst. treatment is to
B. ameloblastoma. A. obtain adequate access.
C. odontogenic keratocyst. B. use a chelating agent.
D. rarefying osteitis. . C. irrigate copiously.
D. use reamers instead of files.
An inflammatory cellular infiltrate found in the
connective tissue of healthy gingiva is Following successful root canal therapy, the most
A. a routine microscopic finding. desirable form of tissue response at the apical
B. an indication of systemic disease. foramen is
C. composed chiefly of macrophages. A. cementum deposition into the apical foramen.
D. a tissue response to food decomposition. B. formation of a connective tissue capsule over
the foramen.
A deflection of the mandible towards the right upon C. proliferation of epithelium from the apical
opening may be due to periodontal ligament.
A. ankylosis of the right temporomandibular joint. D. a chronic low grade inflammatory response
B. internally displaced disk in left surrounded by normal bone.
temporomandibular joint.
C. bilateral fracture of the neck of the condyles. Treatment for a traumatized tooth displaying
D. hypermobility of the right temporomandibular crazing of the enamel is
joint. A. pulpotomy.
B. pulpectomy.
Which of the following should NOT be C. stainless steel crown cemented with calcium
administered to a patient with chest pain hydroxide.
consistent with a myocardial infarction? D. endodontic treatment and stabilization of tooth
A. Epinephrine. with a rigid type splint.
B. Nitroglycerin. E. periodic observation. .
C. Oxygen.
D. Morphine. A patient presents with constant excruciating pain
E. Acetylsalicylic acid. . from a tooth that feels "long" and is sensitive to
pressure. The most likely diagnosis would be a/an
The most common site of a basal cell carcinoma is A. acute pulpitis.
the B. chronic abscess.
A. middle third of the face. C. acute apical periodontitis.
B. lower lip. D. granuloma. .
C. tongue.
D. oral mucosa. . The most efficient cutting instrument used in the
root canal is a
During root canal therapy, which of the following A. barbed broach.
organisms will cause endocarditis in a patient with B. reamer.
valvular heart disease? C. file.
A. Hemolytic streptococcus. D. rat tail file.
B. Non hemolytic streptococcus. E. Hedstrom file. .
C. Coagulase positive staphylococcus.
D. Candida monilia. During endodontic treatment a file is broken. The
E. Bacteroides melaninogenicus. fragment is 3mm long and is lodged tightly in the
apical third of the canal. No radiographic changes
The microorganisms responsible for formation of a at the apex are evident. In addition to informing the
dental or periapical granuloma are found mainly in patient, you would
the A. extract the tooth.
A. granuloma. B. perform an apicoectomy and place a reverse
B. root canal. filling.
C. periapical periodontal ligament. C. resect the apical section of the root containing
D. periapical alveolar bone and bone marrow. the broken instrument.
E. periapical cementum. D. complete the root canal filling and place the
patient on recall.

[Type text]
Sinan Salim

Which of the following is the most probable A patient with a mandibular Kennedy Class I
postoperative complication of bleaching a tooth removable partial denture has returned for the first
that has not been adequately obturated? adjustment. The patient's complaint is that the
A. Fracture. partial denture moves whenever the tongue is
B. Discolouration. placed in either cheek. The most likely cause is
C. Retrograde pulpitis. that
D. Acute apical periodontitis. A. the lingual borders are overextended.
E. External cervical root resorption. B. the lingual borders are too thick.
C. the clasps are too loose and need to be bent
In which of the following defects is "bone-fill" most deeper into the undercut area to increase
likely to occur? retention.
A. One-walled bony defect. D. indirect retention is not adequate.
B. Two-walled bony defect.
C. Combination one and two-wall bony defects. A protrusive relation record should be made by
D. Three-walled bony defect. instructing the patient to protrude the mandible
E. None of the above. . A. 1mm.
B. 3-6mm.
Which of the following factors could cause a C. 10-15mm.
casting to fit tighter in the mouth than on the cast? D. 15-20mm
A. Too much water in mixing the stone for the cast.
B. Too little water in mixing the stone for the cast. The condylar inclination as set on the articulator
C. Duplication impression slightly oversized. from a protrusive record is
D. Improper wax-up of the partial. A. the actual condylar shape.
B. parallel to the Frankfort horizontal plane.
When using alginate impression material, which of C. a mechanical equivalent of the actual condylar
the following statements is correct? slope.
A. Store the impression in water at 37 degrees C D. parallel to the occlusal plane.
prior to pouring the cast. E. the Bennett angle. .
B. Remove the impression slowly from the
undercuts. In complete denture fabrication, increased vertical
C. Control the setting time by changing the dimension of occlusion will result in
water/powder ratio. A. a drooping of the corners of the mouth.
D. Pour the cast immediately upon removal of the B. creases and wrinkles in the region of the mouth.
impression from the mouth. C. trauma to the underlying supporting tissues.
D. poor denture retention.
The best method to control the setting time of an E. a loss of muscle tone.
irreversible hydrocolloid is to alter the
A. temperature of the water. In a distal extension partial denture (free-end
B. water-powder ratio. saddle), the most effective means of limiting
C. mixing time. applied loads to abutment teeth is by
D. composition. A. splinting abutments to adjacent teeth.
B. using the abutment teeth without splinting.
In a removable partial denture, a palatal strap is C. using porcelain replacement teeth.
used instead of a narrow bar because it is D. maintaining a stable base-tissue relationship.
A. more rigid with less apparent bulk.
B. easier to polish. An occlusal rest should be prepared so that
C. more stable. A. occlusal forces are directed perpendicular to the
D. less irritating to the soft tissues. long axis of the tooth.
E. more hygienic. . B. occlusal forces are directed parallel to the long
axis of the tooth.
C. occlusal forces are concentrated near the
marginal ridge.
D. the tooth is prevented from rotating.

The polymerization of methyl methacrylate is


A. endothermic.
B. hydrophilic.
C. hydrolytic.
D. exothermic.

[Type text]
Sinan Salim

In cobalt-chromium alloys, the metal most The most common area for the location of
responsible for corrosion resistance is periapical osteofibrosis (cementoma) is
A. silver. A. maxillary anterior region.
B. nickel. B. mandibular anterior region.
C. cobalt. C. mandibular premolar region.
D. chromium. D. maxillary posterior region.
E. tungsten. . E. All of the above. .

In a fixed bridge, the most favorable ratio for an Which of the following impression materials will
abutment tooth is when the distort the most when stored in air for 24 hours
A. root and anatomic crown are equal in length. before pouring?
B. attached root and clinical crown are equal in A. Condensation cured silicone.
length. B. Polyether.
C. attached root is twice the length of the clinical C. Polysulfide.
crown. D. Irreversible hydrocolloid
D. attached root is half the length of the clinical
crown. In complete denture fabrication, the most important
clinical record is
A patient wearing a maxillary complete denture A. excursive movements.
complains that it is dislodged when he yawns. This B. protrusive relation.
occurs because the C. centric relation.
A. buccal flanges are overextended. D. interdigitation. .
B. anterior teeth are too long.
C. occlusal plane is too high. Assuming that the maxillary cast is already
D. palatal vault is too flat. mounted, the centric relation record is used to
A. determine the vertical dimension of occlusion.
In fixed bridge construction, significant premature B. mount the mandibular cast in the articulator.
occlusal contacts in teeth other than the abutment C. assist in establishing the occlusal plane.
teeth should D. record the inclination of the condyle guidance.
A. be eliminated while the bridge is being made.
B. be eliminated after the bridge has been made. The distal termination of the maxillary complete
C. not be eliminated. denture base is dictated by the
D. be eliminated before the bridge is made. A. tuberosity.
E. None of the above. B. foveae palatinae.
C. posterior palatal seal.
Two of the most common causes of clicking D. vibrating line. .
sounds when speaking with complete dentures are
A. a reduced vertical dimension and improperly In root canal therapy, the most effective irrigating
balanced occlusion. solution for dissolving organic debris is
B. excessive vertical dimension and poor denture A. ethylene diamine tetra acetic acid (EDTA).
retention. B. sodium hypochlorite.
C. use of too large a posterior tooth and too little C. calcium hydroxide.
horizontal overlap. D. hydrogen peroxide.
D. improper relation of teeth in relation to the ridge E. sodium chloride. .
and excessive anterior vertical overlap.
The setting of a zinc-oxyphosphate cement can
A nonrigid connector in a fixed partial denture is best be retarded by
used A. decreasing the particle size.
A. to overcome a problem of parallelism. B. increasing the concentration of water in the
B. when one abutment is periodontally involved. liquid.
C. in long-span bridges. C. reducing the rate of addition of the powder to
D. to avoid too many solder joints. the liquid.
D. increasing the temperature of the mixing slab.
For a porcelain fused to metal restoration, the E. addition of oleic acid.
metal surface
A. requires some degree of mechanical retention.
B. should not be heat treated.
C. requires a well polished surface.
D. must develop an oxide for chemical bonding.
E. should be treated in hydrochloric acid.

[Type text]
Sinan Salim

Three days after cementation of a fixed bridge, The muscles used when closing the jaws to
marked discomfort to heat and cold is most likely maximum intercuspation include
the result of A. medial (internal) and lateral pterygoid,
A. failure to remove excess marginal cement. masseter, geniohyoid.
B. an occlusal prematurity. B. temporalis, medial pterygoid, masseter,
C. devitalization of an abutment tooth. geniohyoid.
D. gingival recession. . C. medial pterygoid, temporalis, masseter.
D. lateral (external) pterygoid, masseter,
In recording centric relation registration, temporalis, geniohyoid.
perforation of the recording material must be
avoided because A primordial cyst
A. the material will undergo dimensional changes. A. develops in place of a tooth.
B. the perforation would not permit an accurate B. attaches to the apex of a tooth.
mounting. C. attaches to the crown of a tooth.
C. contact of teeth could deflect the mandible. D. remains after the tooth is extracted.
D. the recording material will be too weak and may
fracture. An early radiographic sign of chronic periodontitis
is
A patient's four mandibular incisors were A. widening of vascular canals.
traumatized three years ago in an accident. B. "notching" of the crestal lamina dura.
Radiographs now show apical radiolucencies C. enlargement of the interdental medullary
associated with all four teeth. The recommended spaces.
treatment is to D. decreased radiolucency of the interdental
A. test the pulp vitality and perform root canal trabeculae
therapy on teeth with no response.
B. perform root canal therapy and curette the area. Serial extraction for the correction of an
C. extract and place a bonded bridge. orthodontic problem is CONTRAINDICATED in
D. postpone treatment and recheck status patients with
periodically. A. a Class II molar relationship.
B. minimal overbite.
In a young patient, replantation is the treatment of C. minimal overjet.
choice in an accidentally avulsed permanent tooth. D. severe arch crowding.
Ankylosis is seldom a complication following
replantation. Predominant organisms in necrotizing ulcerative
A. The first statement is true, the second gingivitis (NUG) are
statement is false. A. spirochetes and fusiforms.
B. The second statement is true, the first B. Porphyromonas gingivalis.
statement is false. C. Prevotella intermedia.
C. Both statements are true. D. Actinomyces viscosus.
D. Both statements are false.
Endotoxin is
For the initiation of caries, the pH in the bacterial A. a cell wall component of gram-negative
plaque must be bacteria.
A. below 5.5. B. a potent inflammatory agent.
B. above 5.5. C. present in diseased root cementum.
C. above 6.0. D. All of the above.
D. below 4.0.
E. at 6.8. . Benzodiazepines have all of the following actions
EXCEPT
The anticariogenic effect of systemic fluoride is A. muscle relaxation.
related principally to the B. sedation.
A. bacteriocidal action on oral flora. C. amnesia.
B. bacteriostatic action on oral flora. D. anticonvulsant action.
C. buffering effect on acids produced by cariogenic E. analgesia. .
bacteria.
D. alteration in the composition of the enamel. Propylthiouracil is a drug used in the treatment of
A. leukemia.
B. hyperthyroidism.
C. sodium retention.
D. intestinal parasites.
E. allergic manifestations.

[Type text]
Sinan Salim

The rate of set of alginate impression materials Periapical infection from a maxillary permanent
can be increased by first molar may spread by direct extension to which
A. increasing the water/powder ratio. of the following spaces?
B. increasing the temperature of the mixing water. A. Buccal.
C. adding sodium phosphate to the mixing water. B. Infratemporal.
D. decreasing the amount of mixing. C. Submandibular.
D. Pterygomandibular. .
In a normal occlusion the buccal cusps of maxillary
teeth occlude Should post-operative bleeding occur 48 hours
A. with the lingual surface of the mandibular teeth. after a dental extraction, you would suspect
B. in the central fossa of the mandibular teeth. A. decreased production of clotting factors by the
C. with the top of the buccal cusp of the liver.
mandibular teeth. B. the development of circulating inhibitors to
D. with the buccal surface of the mandibular teeth. Factor VIII.
C. a decrease in the serum level of plasmin due to
Which of the following is correct? a deficiency of its precursor plasminogen.
A. Supragingival calculus does not rely on salivary D. heparin release from mast cells in the walls of
minerals for its calcification. the alveolus.
B. Subgingival calculus is not a by-product of E. bacterial streptokinase action on the clot within
streptoccoci mutans. the alveolus.
C. Subgingival calculus is a result rather than an
initiating factor in periodontal disease. Odontogenic infections can spread by
D. All of the above. A. direct extension.
B. lymphatics.
Reciprocation as applied to partial dentures refers C. ingestion.
to the D. All of the above. .
A. function of the occlusal rest to conteract
occlusal forces. A patient has one remaining tooth, the maxillary
B. resistance to flexion of the clasp arm. second molar with diverging roots, which requires
C. return to a passive state of the flexed clasp. extraction. Radiographic examination reveals a low
D. function of the clasp arm to counteract forces lying sinus and very thin alveolar bone. Your
exerted by the retentive clasp arm. approach is
A. routine forcep extraction.
In ideal centric occlusion, the distobuccal cusp B. reflection of mucoperiosteal flap and sectioning
(middle buccal cusp) of the mandibular permanent of the tooth with a rotary instrument prior to
first molar occludes in the removal.
A. central fossa of maxillary first molar. C. splitting of the tooth with an osteotome and
B. mesial triangular fossa of maxillary first molar. mallet followed by the removal of the pieces.
C. distal triangular fossa of maxillary first molar. D. amputation of the crown of the tooth with a
D. occlusal embrasure between maxillary first and rotary instrument leaving the roots in place.
second molars.
A patient on anticoagulant drugs who requires an
A periapical abscess of a mandibular second extraction has a prothrombin time of 20~seconds.
molar that has perforated the lingual cortical plate The control is 15~seconds. You would
below the origin of the mylohyoid muscle may A. administer vitamin K after the extraction.
result in an infection of the B. administer vitamin K before the extraction.
A. buccal space. C. extract the tooth and use local measures to
B. sublingual space. control bleeding.
C. submandibular space. D. discontinue anticoagulation drugs one week
D. submental space. before extraction.
E. superficial masticatory space.
In the etiology of chronic inflammatory periodontal
When a second canal is located in mandibular disease, the primary local factor is
incisors, it is most frequently found A. occlusal trauma.
A. labial to the main canal. B. plaque formation.
B. lingual to the main canal. C. dietary deficiencies.
C. mesial to the main canal. D. age of the patient. .
D. distal to the main canal.
E. None of the above. .

[Type text]
Sinan Salim

Gingival bleeding associated with chronic marginal With excessive forward mandibular growth in a
gingivitis is due to patient with minimal overjet, the mandibular
A. lack of Vitamin C. incisors will most likely
B. destruction of vessels of the periodontal A. tip labially.
ligament. B. tip lingually.
C. excessive pocket depth. C. tip laterally.
D. micro-ulceration of sulcus epithelium. D. remain unchanged. .

Periapical odontogenic cysts are associated with A missing permanent second premolar will likely
A. impacted wisdom teeth. result in
B. congenitally missing teeth. A. normal exfoliation of primary second molar.
C. non-vital teeth. B. delayed exfoliation of primary second molar.
D. a history of traumatic injury. C. early exfoliation of primary second molar.
D. caries development in the primary second
Which of the following changes in colour, contour molar.
and texture are indicative of chronic marginal
gingivitis? The best time to correct a maxillary central incisor
A. Red, swollen, increased stippling. cross-bite is
B. Cyanotic, cleft formation, lack of stippling. A. after the permanent canines erupt.
C. Red, swollen, lack of stippling. B. after the permanent central incisors erupt.
D. Pink, swollen, lack of stippling. C. after the permanent lateral incisors erupt.
D. during the eruptive stage of the central incisors.
The most common clinical sign of primary occlusal
trauma is The orthodontic tooth movement most likely to
A. mobility. show relapse is
B. pocket formation. A. tipping.
C. bone loss. B. rotation.
D. gingival irritation. . C. intrusion.
D. translation. .
Of the following, which has the worst prognosis?
A. Occlusal traumatism. The most frequent cause of tooth loss in the
B. Gingivitis. elderly is
C. Juvenile periodontitis. A. bruxism.
D. Periodontal atrophy. B. caries.
E. Moderate periodontitis. C. periodontal disease.
D. use of a removable partial denture.
Which one of the following oral preventive aids E. extraoral trauma. .
does not remove plaque?
A. Toothbrush. Pre-operative endodontic radiographs will show
B. Dental floss. the
C. Stimudents. A. presence of a pulp exposure.
D. Proxy brush. B. exact location of the apical foramen.
E. Irrigating devices. C. presence of active infection.
D. vitality of the pulp.
In examining edematous gingiva, the tip of the E. size of the pulp chamber and root canal(s).
periodontal probe extends
A. to the cemento-enamel junction. A mucoperiosteal flap which reflects both mucosal
B. to the coronal aspect of subgingival calculus. tissue and periosteum from underlying bone is
C. to the coronal aspect of the epithelial A. split-thickness.
attachment. B. full thickness.
D. to the alveolar bone crest. C. partial thickness.
E. apical to the epithelial attachment. D. mucosal. .

A malocclusion characterized by labioversion of At what age is a child expected to have 12 erupted


maxillary lateral incisors and deep overbite typifies primary teeth and 12 erupted permanent teeth?
Angle's classification A. 4 1/2 years.
A. I. B. 6 1/2 years.
B. II, Division 1. C. 8 1/2 years.
C. II, Division 2. D. 11 1/2 years.
D. III. E. None of the above.

[Type text]
Sinan Salim

Contraction of the external (lateral) pterygoid Which of the following has the strongest analgesic
muscle pulls the disc of the temporomandibular properties?
joint A. Acetylsalicylic acid (325mg).
A. forward and medially. B. Ibuprofen (400mg).
B. backward and medially. C. Codeine (15mg).
C. forward and laterally. D. Acetaminophen (300mg).
D. backward and laterally.
E. None of the above. The mucogingival line denotes the
A.
In periodontitis, the associated bacteria are B. separation between the marginal gingiva and
primarily located within the the alveolar mucosa.
A. free gingiva. C. separation between the attached gingiva and
B. gingival sulcus. the marginal gingiva.
C. alveolar bone. D. junction between the attached gingiva and the
D. periodontal membrane. alveolar mucosa.
E. junction between the mucoperiosteum and
Which of the following drugs has/have sedative gingiva.
properties?
A. Benzodiazepines. Periodontitis
B. Barbiturates. 1. develops from gingivitis.
C. Meperidine. 2. is associated with continuous destruction of the
D. All of the above. . alveolar bone.
3. goes through stages of tissue destruction and
In complete denture construction, the physiologic quiessence.
rest position 4. results in occlusal traumatism.
A. provides a guide to establish the vertical
dimension of occlusion. A. (1) (2) (3)
B. determines the level of the occlusal plane. B. (1) and (3)
C. provides a guide for the selection of cusp C. (2) and (4)
inclination. D. (4) only
D. determines the shape of the compensating E. All of the above. .
curve.
E. is useful to the determination of condylar In a teenager with aggressive (juvenile)
inclination. periodontitis, the predominant microorganism at
the base of the pockets is
The luting materials that will bond to enamel are A. Porphyromonas gingivalis.
1. zinc phosphate. B. a fusospirochete.
2. polycarboxylate. C. Actinobacillus actinomycetemcomitans.
3. zinc silicophosphate. D. Staphylococcus aureus.
4. glass ionomer.
A suprabony pocket may be associated with:
A. (1) (2) (3) A. increase in sulcular depth.
B. (1) and (3) B. loss of attachment with accompanied horizontal
C. (2) and (4) bone loss.
D. (4) only C. loss of attachment with accompanied vertical
E. All of the above. . bone loss.
D. loss of attachment without accompanied
Which of the following should NOT be prescribed increase in sulcular depth.
for a patient receiving warfarin (Coumadin)? E. A. and B. .
1. Acetylsalicylic acid.
2. Oxycodone. Maxillary removable partial denture major
3. Ketorolac. connectors should be 3-6mm from the marginal
4. Codeine. gingiva in order to
A. minimize food impaction.
A. (1) (2) (3) B. reduce irritation of the marginal gingiva.
B. (1) and (3) C. allow sufficient length for the minor connectors.
C. (2) and (4) D. facilitate movement of the tongue and allow
D. (4) only acceptable phonetics.
E. All of the above. E. prevent rotation in an anterior-posterior direction
of the major connector.

[Type text]
Sinan Salim

The healthy junctional epithelium (epithelial Based on the results of this study, which of the
attachment) following recommendations would you make to
A. is keratinized. patients in your practice:
B. has a granular layer. A. Use of LA as supplement to regular daily oral
C. is permeable to tissue fluids. hygiene measures to control gingivitis.
D. contains lymphocytes. B. Since LA and PX are equally effective, either
can be used with comparable efficacy for the
An 8-year old patient with all primary molars still control of gingivitis.
present exhibits a cusp-to-cusp relationship of C. The results are not conclusive enough to
permanent maxillary and mandibular first molars. recommend the use of either LA or PX for
The management of this patient should be to gingivitis.
A. plan serial extractions for more normal
adjustment of the occlusion. For amalgam restorations, a 90 cavosurface
B. refer the patient to an orthodontist for angle accommodates the
consultation. 1. condensing of amalgam.
C. place a cervical headgear to reposition 2. compressive strength of amalgam.
maxillary molars. 3. tensile strength of amalgam.
D. disk the distal surfaces of primary mandibular 4. compressive strength of enamel.
second molars to allow normal adjustment of
permanent molars. A. (1) (2) (3)
E. observe. B. (1) and (3)
C. (2) and (4)
A major disadvantage of the cervical headgear D. (4) only
used for some orthodontic treatment is the risk of E. All of the above.
A. intrusion of maxillary canines.
B. extrusion of maxillary incisors. The problem most likely to result from a temporary
C. extrusion of maxillary molars. crown with inadequate proximal contacts is:
D. deformity of the neck. A. Frequent decementation of the temporary
E. psychological trauma due to appearance. crown.
B. Hyperplastic papillae.
Hydroxyapatite C. Difficulty in seating the permanent crown.
1. can be used to eliminate osseous undercuts. D. An esthetically compromised restoration.
2. is derived from coral.
3. is biocompatible. Polyethers are NOT recommended for impressions
4. is resistant to fracture. in mouths where periodontal disease has left wide
5. is osteogenic. interdental embrasures because they have
A. high elastic modulus.
A. (1) (3) (5) B. low tear strength.
B. (1) (2) (3) C. high viscosity.
C. (2) (4) (5) D. poor elastic memory. .
D. All of the above. .
What is the most important mechanical property to
The abstract indicates that the study being consider in the selection of an alloy for a long and
reported narrow porcelain fused to metal bridge?
1. attempted to minimize subject selection bias. A. Elastic modulus.
2. attempted to minimize measurement bias. B. Proportional limit.
3. controlled variables which could affect the C. Toughness.
results (confounders). D. Strength. .
4. should be generalizable to the general adult
population. Retentive pins
A. (1) (2) (3) A. assist in preventing shearing of non-restored
B. (1) and (3) cusps.
C. (2) and (4) B. decrease the compressive strength of the
D. (4) only amalgam.
E. All of the above. C. increase the tensile strength of the amalgam.
D. provide resistance form to the preparation.
E. decrease microleakage by bonding with the
amalgam.

[Type text]
Sinan Salim

What type of tissue covers the articular surfaces of The maxillary permanent canine erupts
the human temporomandibular joint? A. before the mandibular permanent canine.
A. Hyaline cartilage. B. before the maxillary first premolar
B. Fibrocartilage. C. before the maxillary second permanent molar.
C. Epithelium. D. after the maxillary second permanent molar.
D. Dense, avascular, fibrous connective tissue. E. None of the above. .
E. Loose, vascular, fibrous connective tissue.
The most common initial sign of occlusal trauma is
When removing bone or sectioning roots of teeth A. tooth mobility.
with a high-speed handpiece, the air/water B. tooth sensitivity.
combination should be set with C. radiographic evidence of increased periodontal
A. air and water on. space.
B. water only. D. loss of pulp vitality. .
C. air only.
D. neither air nor water. The radiographic appearance of vertical (angular)
bone loss is characterized by an
The white appearance of the oral mucosa seen alveolar crest which is
following extended local application of aspirin is A. not parallel to an imaginary line drawn between
the result of the cemento-enamel junctions of adjacent teeth.
A. hyperparakeratosis. B. 0-2mm apical to the cemento-enamel
B. acanthosis. junction.
C. coagulation necrosis. C. 2-3mm apical to the cemento-enamel
D. edema. . junction.
D. more than 4mm apical to the cemento-enamel
A 4mm diameter carious exposure occurs on a junction.
permanent first molar of a 7 year old child. The
tooth is vital and has no periapical involvement. In a healthy patient with an acute localized
The appropriate initial treatment would be to periodontal abscess, initial treatment must include
perform a/an A. scaling and root planing.
A. pulp capping. B. occlusal adjustment.
B. pulpotomy. C. prescription of an antibiotic.
C. pulpectomy. D. prescription of an analgesic
D. extraction. .
When making a reline impression for a distal
While removing an impression from the oral extension partial denture (Kennedy Classification
cavity, some material falls in the throat. The 1), the finger pressure should be applied to
patient coughs and breathes. You should A. the occlusal surfaces of the denture teeth.
A. push once on the abdomen at the B. the rest areas and indirect retainer.
level of the umbilicus. C. the denture flanges.
B. push 6 to 10 times on the abdomen at D. the clasps.
the level of the diaphragm.
C. slap the patient in the back between Which of the following conditions is the most
the shoulder blades. dangerous?
D. make the patient drink cold water A. Acute periapical abscess of a mandibular
without breathing. central incisor.
E. instruct the patient to continue B. Middle face cellulitis.
coughing. C. Chronic periapical abscess of a mandibular
third molar.
Osteosarcoma differs from fibrous dysplasia D. Infected dentigerous cyst.
because it
A. can invade soft tissue. A 33 year old patient presents with an acute
B. has ill-defined radiographic margins. infection of tooth 2.3 that has resulted in mild
C. can be radiolucent, mixed or radiopaque. cellulitis of the left cheek. His medical history reveals
D. is difficult to curette from normal bone. that he has had an anaphylactic reaction to
amoxicillin in the past. Which of
Pain, associated with temporomandibular disorders is the following drugs is CONTRAINDICATED
most frequently caused by in the management of this patient?
A. impacted third molars. A. Clindamycin.
B. perforation of the articular disks. B. Cephalexin.
C. muscle hyperactivity. C. Erythromycin.
D. subluxation. D. Metronidazole.

[Type text]
Sinan Salim

Local anesthesia of the infraorbital nerve will Periapical cemental dysplasia is


anesthetize which of the following groups? A. painful.
A. Tip of the nose, lower eyelid, upper lip. B. expansile.
B. Tip of the nose, skin of the cheek, upper lip. C. associated with vital teeth.
C. Side of the nose, lower eyelid, upper lateral D. premalignant. .
incisor.
D. Side of the nose, lower eyelid, upper lip. Which of the following is one of the principle jaw-
opening muscles?
A patient has valvular heart disease and is allergic A. Temporalis.
to penicillin. The most appropriate antibiotic to be B. Lateral (external) pterygoid.
used prophylactically is C. Masseter.
A. erythromycin. D. Medial pterygoid. .
B. metronidazole.
C. amoxicillin. Which of the following is NOT characteristic of
D. tetracycline. acute leukemia?
E. clindamycin. . A. Gingival enlargement.
B. Decreased bleeding time.
The outline form for an amalgam restoration in pit C. Elevated leukocyte count.
and fissure cavities is controlled by all of the D. Anemia.
following EXCEPT the E. Thrombocytopenia.
A. extent to which the enamel has been involved
by the carious process. Following injection of a local anesthetic, a healthy
B. lateral spread of caries along the dentino- patient suddenly experiences difficulty breathing.
enamel junction. You detect a rash and swelling of the face and
C. extension that must be made along the fissures neck. The most appropriate drug to administer first
in order to achieve sound and smooth margins. is
D. need to terminate the margins on a cusp ridge A. hydroxycine.
or marginal ridge crest. B. epinephrine
E. extent of undermining of the enamel by the C. hydrocortisone.
carious process. D. diphenhydramine.

Which of the following fibre groups of the Reduction of arch length most frequently follows
periodontal ligament is responsible for support the premature loss of the primary
against masticatory forces? A. maxillary first molar.
A. Horizontal. B. mandibular second molar.
B. Dentoperiosteal. C. mandibular canine.
C. Oblique. D. maxillary canine. .
D. Gingival.
A squamous cell carcinoma of the midline anterior
The periodontal condition found in 14 to 19- year floor of the mouth normally spreads to the
olds exhibiting localized advanced vertical bone A. unilateral neck lymph nodes.
loss involving the first molars and the incisors is B. mediastinal lymph nodes
A. desquamative gingivitis. C. bilateral neck lymph nodes.
B. acute necrotizing ulcerative gingivitis. D. retropharyngeal lymph nodes
C. juvenile periodontitis.
D. rapidly progressive periodontitis. A pale 8 year old patient presents with generalized
E. advanced destructive chronic periodontitis. gingival enlargement and spontaneous bleeding.
The most appropriate initial management of this
Fordyce's spots or granules have the clinical patient is to
appearance of small yellow spots on the buccal A. perform an incisional biopsy.
mucosa. This is due to the presence of B. obtain a cytologic smear.
A. sweat glands. C. order a complete blood count with a differential.
B. implanted epithelium. D. obtain bacterial cultures.
C. cyst formation. E. order fasting blood glucose levels.
D. sebaceous glands.
E. hyperkeratosis. .

[Type text]
Sinan Salim

Epinephrine, when used to treat an anaphylactic The best way for a dentist to ensure efficacy in a
reaction, has the following desirable effects disinfection solution is to
EXCEPT A. make a fresh solution every day.
A. rapid onset. B. follow the manufacturers instructions.
B. vasopression. C. increase the concentration of the product.
C. bronchodilation. D. increase the time of contact with the product.
D. interference with AB/Ag reaction.
Oral hygiene for infants teeth should begin when
Compared to unstimulated saliva, stimulated saliva A. the first primary molars erupt.
is more beneficial for oral health because of a B. all primary teeth erupt.
higher C. the first tooth erupts into the oral cavity.
A. calcium level. D. the infant is weaned from the nursing bottle or
B. phosphate level. breast.
C. mucin level
D. pH level. . A large, deeply furrowed tongue is commonly
found in patients with
In a peripheral nerve such as the trigeminal, A. Pierre Robin Syndrome.
unmyelinated C type fibres convey B. geographic tongue.
A. muscle spindle information. C. ectodermal dysplasia.
B. reflexes such as the jaw jerk reflex. D. Downs Syndrome.
C. tendon organ information.
D. chronic pain information. The most likely cause of fracture of a porcelain
E. reflexes such as the jaw opening veneer, that leaves the underlying metal exposed,
reflex. on a porcelain fused to metal bridge is
The most effective method of destroying micro- A. flexure of the bridge under the occlusal load.
organisms on dental instruments is B. a porcelain veneer that is more than 2mm thick.
A. a 10 minute immersion in boiling water. C. a contamination of the metal prior to porcelain
B. a 15 minute immersion in disinfectant solution at application.
room temperature. D. porosities inside the veneer.
C. a 15 minute exposure to wet heat in an oven at E. overglazing of the porcelain.
o
100 C.
D. a 20 minute exposure to dry heat in an oven at In the maxilla, infection from which of the following
o
120 C. teeth may drain through the palatal plate of bone
o
E. 20 minutes in an autoclave at 120 C and present as a palatal abscess?
Which type of bacteria are used as a control to 1. Lateral incisors.
show the effectiveness of sterilization? 2. First premolars.
A. Gram-negative. 3. First molars.
B. Anerobic. 4. Third molars.
C. Spores. A. (1) (2) (3)
D. Oral. . B. (1) and (3)
C. (2) and (4)
During the extraction of an impacted tooth 3.8 the D. (4) only
lingual nerve is damaged. All of the following can E. All of the above. .
occur EXCEPT a
A. loss of taste from the anterior 2/3 on the left In dental radiography, the lamina dura is
side of the tongue. A. a radiolucent line around roots.
B. deviation of the tongue to the left on protrusion. B. a radiopaque line around roots.
C. decreased salivary output from the left C. an irregular radiolucent line around the roots.
sublingual gland. D. None of the above. .
D. decreased salivary output from the left
submandibular gland. When bonding composite resin to a tooth
E. numbness of the floor of the mouth on the left preparation that includes both enamel and dentin,
side the term total etching means that
A. all of the tooth is etched.
Of the following, which source is the most credible B. only the enamel and dentin involved with the
for updating dental knowledge? tooth preparation are etched.
A. Dental supply publications. C. only the enamel involved with the preparation is
B. Local professional journals. etched.
C. Review articles from scientific journals. D. only the dentin involved with the preparation is
D. Research articles from scientific journals. etched.

[Type text]
Sinan Salim

Following the administration of a right inferior


Exfoliative cytology can be useful in detecting viral alveolar nerve block, right facial paralysis is noted.
cytopathologic effects in all of the following This condition was caused by inadvertent injection
EXCEPT into the
A. shingles. A. pterygopalatine fossa.
B. cold sores (herpes labialis). B. facial canal.
C. chicken pox. C. parotid gland.
D. hairy leukoplakia. D. submandibular region.
E. acute herpetic gingivostomatitis E. sublingual gland.

If hypothyroidism occurs in the adult, it is The immediate management of a hematoma


associated with following a posterior superior alveolar nerve block
A. exophthalmos. is to
B. weight loss. A. incise into the buccal sulcus.
C. generalized edema. B. apply firm direct pressure.
D. tachycardia. C. aspirate with a needle.
E. mental defects. D. place hot towels over the cheek

A 9 year old boy requires treatment following a What is the most appropriate medication to
facial injury resulting in a fracture of a maxillary manage post-operative pain for an asthmatic
central incisor that involves the enamel only. The patient taking beclomethasone (Beclovent) and
tooth tests negative to an electric pulp tester. This salbutamol (Ventolin)?
finding indicates that the tooth A. Acetylsalicylic acid.
A. is non-vital and should be extracted. B. Acetylsalicylic acid/codeine combination.
B. is non-vital and endodontic therapy is indicated. C. Ibuprofen.
C. has a root fracture and should be extracted. D. Acetaminophen/codeine combination.
D. should be observed and tested again at a later E. Naproxen. .
date.
In a Class I occlusion the
Tripoding of casts serves as a useful method for A. distal inclined plane of the maxillary canine
A. determining the position for precision articulates with the mesial inclined plane of the
attachments. mandibular canine.
B. locating unfavorable tissue undercuts. B. mesial inclined plane of the maxillary canine
C. locating the height of contour. articulates with the distal inclined plane of the
D. accurately returning the cast to the surveyor. mandibular canine.
C. the primary canines are end-to-end.
Which of the following is a disease characterized D. the permanent canines are end to end.
by a general lack of bone marrow activity?
A. Thalassemia. In a dental resin composite, the silane coupling
B. Polycythemia. agent provides a chemical bond between the
C. Aplastic anemia. A. filler particles and the matrix.
D. Pernicious anemia. B. filler particles and the tooth structure.
E. Sickle cell anemia. . C. composite and the adhesive resin.
D. hybrid layer and the composite.
Periodontitis
1. develops from gingivitis. The inverse square law states that if 100 x-ray
2. goes through stages of activity and remission. photons are measured 1m from a point source of
3. is associated with bone loss. radiation, the number of photons measured at 2m
4. is caused by occlusal trauma. from the same source would be
A. 50.
A. (1) (2) (3) B. 33.
B. (1) and (3) C. 25.
C. (2) and (4) D. 10.
D. (4) only
E. All of the above. . Which of the following tissues is the LEAST
sensitive to the effects of x-radiation?
A. Hematopoietic.
B. Gonadal.
C. Bone.
D. Glandular.

[Type text]
Sinan Salim

The most common cause of bilateral swelling of Opioids would NOT be indicated for the
the parotid glands in children is management of
A. Mikulicz' disease. A. pain.
B. mumps or acute infectious parotitis. B. severe cough.
C. mixed salivary tumor (pleomorphic adenoma). C. diarrhea.
D. sialolithiasis. D. depression.

The essential clinical sign of periodontitis is The use of polyether impression materials is NOT
A. attachment loss. recommended for full arch impressions of dentate
B. bleeding on probing. patients because they
C. change in gingival colour. A. exhibit viscoelasticity.
D. increased tooth mobility. B. exhibit a high elastic modulus.
C. are thixotropic.
At the try-in appointment, an all metallic crown has D. are hydrophilic
an open margin. The most likely cause is
A. supra occlusion of the crown. For a porcelain fused to metal restoration, the
B. excessive convergence of the axial walls. metal surface
C. excessive thickness of the die spacer. A. requires some degree of mechanical retention.
D. undercontour of the provisional restoration B. should not be heat treated.
interproximally. C. requires a well polished surface.
D. must develop an oxide for chemical bonding.
Composite resin is a satisfactory core build-up
material for endodontically treated teeth to be A factor that increases caries risk in dentate
crowned provided elderly patients is
A. the resin has a high contrast colour with tooth A. age-related decreased salivary flow rate.
structure. B. change in cementum composition.
B. there is an adequate ferrule. C. age-related change in enamel composition.
C. the resin is autopolymerizing. D. presence of chronic periodontal disease.
D. subsequent crown margins are not located on
cementum Which of the following would be considered normal
blood pressure for a healthy 75 year old?
The tissue which is most sensitive to radiation is A. 175/95.
the B. 138/86.
A. nerve. C. 130/100.
B. dental pulp. D. 185/94.
C. lymphoid.
D. muscle. Which of the following would be considered normal
blood pressure for a healthy 75 year old?
Ludwigs angina initially involves a massive A. 175/95.
infection of the B. 150/86.
A. parapharyngeal and retropharyngeal spaces. C. 130/100.
B. submandibular and sublingual regions. D. 185/94.
C. buccal superficial fascial and canine spaces.
D. maxillary and ethmoidal sinuses. Which of the following describes reversible
pulpitis?
A rubber dam in which there is leakage A. Pain to hot.
interproximally has B. Pain to cold.
A. holes punched too far apart. C. Pain to percussion.
B. holes punched too close together. D. No spontaneous pain.
C. too broad a rubber dam arch form.
D. too much tension on the rubber dam holder. When preparing a posterior tooth for an extensive
amalgam restoration, a pin hole preparation should
Which therapeutic agent is absolutely be placed
CONTRAINDICATED in the treatment of herpes A. perpendicular to the pulpal floor.
simplex? B. parallel to the contour of the final restoration.
A. Vitamin C. C. angled 30o away from the pulp chamber.
B. Acetylsalicylic acid. D. parallel to the external root contour.
C. Antibiotics.
D. Corticosteroids.
E. Local anesthetics.

[Type text]
Sinan Salim

All of the following are signs of occlusal trauma Which of the following statements is correct with
EXCEPT respect to zinc containing amalgams?
A. tooth mobility. A. High copper amalgam restorations containing
B. tooth sensitivity. zinc demonstrate better overall survival rates.
C. radiographic evidence of increased periodontal B. Amalgams containing zinc should be used when
space. contamination with moisture is unavoidable during
D. loss of pulp vitality condensation.
C. Zinc is added during the manufacturing
Which of the following is characteristically found in operation to increase the solubility of tin in silver.
Paget's disease? D. Amalgams containing zinc produce a
A. Increased acid phosphatase. significantly better seal than zinc free amalgams.
B. Increased alkaline phosphatase.
C. Osteoporosis. 0.12% chlorhexidine mouthrinses are effective in
D. Incomplete fusion of cranial sutures. modifying plaque quantity and quality because of
their selective activity against
Which of the following is/are (an) indication(s) for A. streptococcus sanguis.
the removal of impacted mandibular third molars? B. lactobacillus acidophilus.
1. Recurrent pericoronitis. C. streptococcus mutans.
2. Prevention of crowding of mandibular incisors. D. lactobacillus casei.
3. Pain.
4. They are impacted. A patient suddenly develops a swelling of the left
A. (1) (2) (3) face and neck with crepitation during the
B. (1) and (3) preparation of a subgingival Class V preparation
C. (2) and (4) on tooth 3.6. The most probable diagnosis is a/an
D. (4) only A. anaphylactoid reaction.
E. All of the above. B. hematoma.
C. cervicofacial emphysema.
Which of the following medications is D. type 1 allergic reaction.
CONTRAINDICATED in the management of a
patient who is taking Coumadin? A 50 year old woman has a history of rheumatoid
A. Acetaminophen. arthritis, bilateral enlargement of one or more
B. Penicillin. salivary glands and lacrimal glands, as well as
C. Acetylsalicylic acid. dryness of the eyes, nose, mouth and throat. The
D. Codeine. diagnosis is
E. Clindamycin. A. erythema multiforme.
B. Reiter's syndrome.
To maintain pulp health and vitality, the preferred C. Gardner's syndrome.
treatment for asymptomatic teeth with very deep D. Sjgren's syndrome.
dentinal caries is E. Plummer-Vinson syndrome.
A. direct pulp capping using a dentin bonding
agent. When epinephrine 1:1000 is administered
B. to avoid pulp exposure using indirect pulp intramuscularly for the management of
capping. anaphylaxis in an adult, the most
C. direct pulp capping using calcium hydroxide. appropriate volume for an initial dose is
D. to remove all affected dentin A. 0.04ml.
B. 0.3 0.5ml.
Polymerization shrinkage in a composite resin is C. 1.0 1.5ml.
reduced by D. 1.8ml.
A. placing a glass ionomer liner on all exposed
dentin before placing composite resin. A 17 year old male patient exhibits delayed
B. doubling the curing time of the resin in eruption of permanent teeth. Radiographs
preparations that are deep. indicate multiple, impacted permanent teeth.
C. using a flowable composite on the gingival floor The head size appears large with prominent
of Class II preparations. frontal eminences and slightly constricted
D. incremental placement of no more than 2mm facial features. The most likely diagnosis is
thickness of composite resin. A. osteopetrosis.
B. osteitis deformans.
C. Gardner's syndrome.
D. acromegaly.
E. cleidocranial dysostosis.

[Type text]
Sinan Salim

Hypercementosis of tooth roots, replacement During root canal therapy, which of the
of skeletal bone with poorly mineralized bone, following organisms will cause endocarditis
and an increased serum alkaline phosphatase in a patient with valvular heart disease?
are characteristic features of A. Hemolytic streptococcus.
A. acromegaly. B. Non hemolytic streptococcus.
B. osteopetrosis. C. Coagulase positive staphylococcus.
C. osteitis deformans (Paget's disease). D. Candida monilia.
D. osteomyelitis. E. Bacteroides melaninogenicus.
E. myelomatosis.
The microorganisms responsible for formation
A differential diagnosis for gingival of a dental or periapical granuloma are found
enlargement should include which of the mainly in the
following conditions? A. granuloma.
A. Multiple myeloma. B. root canal.
B. Monocytic leukemia. C. periapical periodontal ligament.
C. Erythema multiforme. D. periapical alveolar bone and bone
D. Pemphigus vulgaris. marrow.
E. periapical cementum.
A 22 year old patient has been experiencing
general malaise, fever, sore throat and coughing Which structure is responsible for root
for one week. There are multiple ulcerations of the development?
oral mucosa, crusting of the lips and red circular A. Dental papilla.
lesions on the palms of the hands. The most likely B. Hertwig's epithelial sheath.
diagnosis is C. Enamel organ.
A. gonorrhea. D. Cellular cementum.
B. infectious mononucleosis. E. Undifferentiated mesenchymal cells.
C. acute herpetic gingivostomatitis.
D. AIDS. In permanent teeth, two pulp canals are most
E. erythema multiforme. commonly found in the
A. distobuccal root of maxillary molars.
Which of the following is a sign of an B. distal root of mandibular first molars.
acute overdose of acetaminophen? C. palatal root of maxillary first
A. Bronchoconstriction. premolars.
B. Gastric bleeding. D. mesial root of mandibular first
C. Hepatotoxicity. molars.
D. Renal damage.
E. Severe hypotension. A positive and prolonged reaction to a heat
stimulus indicates that the pulp is
Which of the following is the most A. necrotic.
appropriate management for a dry B. in an early hyperemic state.
socket? C. normal.
A. Hydrogen peroxide irrigation of the D. irreversibly damaged.
socket.
B. Vigorous curettage of the socket. A patient presents with constant excruciating
C. Placement of a dressing in the socket. pain from a tooth that feels "long" and is
D. A prescription for antibiotics sensitive to pressure. The most likely
diagnosis would be a/an
Which teeth with pulpitis most often refer A. acute pulpitis.
pain to the ear? B. chronic abscess.
A. Maxillary molars. C. acute apical periodontitis.
B. Mandibular molars. D. granuloma.
C. Maxillary premolars.
D. Mandibular premolars. Complete calcification of the pulp with
obliteration of the pulp chamber may be the
result of
A. a deep carious lesion.
B. a deep composite restoration.
C. traumatic injury.
D. chronic thermal irritation.

[Type text]
Sinan Salim

The permanent first molars of a 7 year old Untreated diabetes mellitus characteristically
patient have pronounced, deep occlusal demonstrates
fissures that are stained. Bitewing radiographs A. hypoglycemia.
show a normal dentino-enamel junction. The B. hyperglycemia.
treatment of choice is C. hypophagia.
A. conservative amalgam restorations. D. hyperlipidemia.
B. glass ionomer restorations. E. dysuria.
C. application of fissure sealants.
D. topical fluoride application. Of the following, the most common site of a
E. no recommended treatment. basal cell carcinoma is the
A. middle third of the face.
Decreased size or obliteration of pulp B. lower lip.
chambers and canals is diagnostic of C. tongue.
A. Hand-Schuller-Christian disease. D. oral mucosa.
B. cleidocranial dysostosis.
C. amelogenesis imperfecta. The most serious complication of a
D. osteogenesis imperfecta. therapeutic dose of radiation to the mandible
is
In a normal occlusion the buccal cusps of A. skin erythema.
maxillary teeth occlude B. osteoradionecrosis.
A. with the lingual surface of the C. obliterating endarteritis.
mandibular teeth. D. loss of hair.
B. in the central fossa of the mandibular E. hyperpigmentation of skin.
teeth.
C. with the top of the buccal cusp of the Asymptomatic, bilateral, reticular, radiating,
mandibular teeth. grayish-white linear lesions of the buccal
D. with the buccal surface of the mucosa most likely represent
mandibular teeth. A. lichen planus.
B. white sponge naevus.
Thinned cortical bone, decreased cancellous C. pemphigus.
trabeculation, enlargement of the medullary D. discoid lupus erythematosus.
cavity and decreased bone density are E. erythema multiforme.
radiographic features of
A. osteomalacia. Occasionally, mucous glands are seen in the
B. osteopetrosis. epithelial lining of a dental cyst. Which of the
C. primary occlusal traumatism. following terms best designates this state?
D. osteoradionecrosis. A. Anaplasia.
E. osteoporosis B. Metaplasia.
C. Dysplasia.
After ossification of the epiphyseal cartilages, D. Neoplasia.
an overproduction of the growth hormone of E. Hyperplasia
the anterior pituitary gland produces the
clinical condition known as Which of the following is characteristically
A. gigantism. found in Paget's disease?
B. acromegaly. A. Increased acid phosphatase.
C. myxoedema. B. Increased alkaline phosphatase.
D. Paget's disease. C. Osteoporosis.
D. Incomplete fusion of cranial sutures.
Routine radiographs of a middle-aged black
female reveal a radiolucency at the apices of A 17 year old male patient exhibits delayed
the mandibular central incisors. The teeth are eruption of permanent teeth. Radiographs
vital. The most likely clinical diagnosis is indicate multiple, impacted permanent teeth.
a/an The head size appears large with prominent
A. apical periodontal cyst. frontal eminences and slightly constricted
B. periapical granuloma. facial features. The most likely diagnosis is
C. periapical cemental dysplasia. A. osteopetrosis.
D. ameloblastoma B. osteitis deformans.
C. Gardner's syndrome.
D. acromegaly.
E. cleidocranial dysostosis.

[Type text]
Sinan Salim

Ocular lesions are associated with When tumour cells revert to a more primitive,
A. lichen planus. embryonic, or undifferentiated form with an
B. herpangina. increased capacity for reproduction and a
C. necrotizing ulcerative gingivitis. decreased function, this is called
D. leukoplakia. A. anaplasia.
E. cicatricial pemphigoid. B. metaplasia.
C. hypoplasia.
In radiography, if target-skin distance is D. hyperplasia.
doubled, the exposure time must be
A. doubled. A constant finding in the radicular cyst is
B. tripled. A. cholesterol clefts.
C. quadrupled. B. foam cells.
D. increased 10 times. C. lining epithelium.
D. polymorphonuclear leukocytes.
Routine radiographs of a middle-aged black E. multinucleated giant cells.
female reveal a radiolucency at the apices of
the mandibular central incisors. The teeth are Increased tooth mobility and the absence of
vital. The most likely clinical diagnosis is lamina dura are signs of
a/an A. hyperthyroidism.
A. apical periodontal cyst. B. hyperpituitarism.
B. periapical granuloma. C. hyperparathyroidism.
C. periapical cemental dysplasia. D. scleroderma
D. ameloblastoma.
Multiple giant cell lesions of the bone are
The most serious complication of a therapeutic associated with
dose of radiation to the mandible is A. hyperthyroidism.
A. skin erythema. B. hypothyroidism.
B. osteoradionecrosis. C. hyperparathyroidism.
C. obliterating endarteritis. D. hypoparathyroidism.
D. loss of hair.
E. hyperpigmentation of skin. Which of the following cysts contains the
crown of a tooth?
Unilateral numbness of the chin is associated A. Radicular.
with B. Multilocular.
A. malignancy. C. Dentigerous.
B. Bell's palsy. D. Inclusion.
C. periapical abscess.
D. trigeminal neuralgia. A positive Nikolsky's sign is a diagnostic
feature of
On a bitewing radiograph of posterior teeth, A. lichen planus.
which of the following is most likely to be B. erythema multiforme.
misdiagnosed as proximal caries? C. pemphigus.
A. Cemento-enamel junction. D. chronic marginal gingivitis.
B. Marginal ridge. E. lupus erythematosus.
C. Carabelli cusp.
D. Calculus. A 14-year old boy presents with bilateral
E. Cemental tear. pearly-white thickening of the buccal mucosa
which has been present since birth. His
A patient with myxedema is characterized by brother has similar lesions. Your diagnosis is
A. exophthalmos. A. leukoplakia.
B. satin skin and fine hair. B. lichen planus.
C. enlargement of the thyroid gland. C. mucous patches.
D. being dull, slow moving, expressionless. D. white-sponge naevus.
E. gross coarse features and an alert look.
Granulation tissue which extends coronally
from the pulp of a carious tooth is known as
A. a pyogenic granuloma.
B. a pulp polyp.
C. epulis granulomatosum.
D. a fibroma.

[Type text]
Sinan Salim

Mottled enamel is a/an A circumscribed radiolucent lesion associated with


A. form of enamel hypoplasia. a retained root tip should be curetted because
B. manifestation of fluorosis. A. pus in the lesion will infect the blood
C. enameloma. clot.
D. side effect of tetracycline therapy. B. the fibrous tissue of the lesion will
prevent new bone formation.
A radicular cyst C. the capillaries of the lesion will cause
A. enlarges rapidly. a bleeding problem.
B. infiltrates bone. D. the lesion may undergo cystic
C. contains fluid. degeneration.
D. cannot cause cortical bone
expansion. The skin lesion associated with actinomycosis
E. is associated with a vital tooth. may be
A. indurated.
A 4-year old child has a normal complement B. fistulated.
of deciduous teeth, but in appearance they are C. purplish-red in colour.
grayish and exhibit extensive occlusal and D. All of the above.
incisal wear. Radiographic examination
indicates some extensive deposits of A benign cementoblastoma is a solitary
secondary dentin in these teeth. This A. circumscribed radiopacity involving
condition is typical of a mandibular molar.
A. cleidocranial dysplasia (dysostosis). B. circumscribed radiolucency
B. amelogenesis imperfecta. involving the apices of the
C. neonatal hypoplasia. mandibular incisors.
D. dentinogenesis imperfecta. C. radiolucency involving the apices of
the maxillary incisors.
Myxoedema is associated with D. unilocular radiolucency in an
A. insufficient parathyroid hormone. edentulous area.
B. excessive parathyroid hormone.
C. insufficient thyroid hormone. A patient having a white blood count of
D. excessive thyroid hormone. 30,000 per cubic millimeter would be said to
have
Which of the following conditions has the A. leukocytosis.
highest white blood cell count? B. leukopenia.
A. Acute myeloblastic leukemia. C. neutrophilia.
B. Polycythemia vera. D. erythropenia.
C. Aplastic anemia. E. erythrocytosis.
D. Infectious mononucleosis.
A sarcoma is a
A periapical granuloma A. benign neoplasm of connective
A. erodes rapidly through bone. tissue.
B. is asymptomatic. B. malignant neoplasm of connective
C. is intensely painful. tissue.
D. occurs only in young adults. C. benign neoplasm of epithelial tissue.
E. occurs at the apex of a vital tooth. D. malignant neoplasm of epithelial
tissue.
The most common complications caused
by supernumerary teeth are Osteoporosis may result from
1. malposition of permanent teeth. A. hypothyroidism.
2. non-eruption of permanent teeth. B. acromegaly.
3. a dentigerous cyst. C. diabetes.
4. an apical periodontal cyst (radicular cyst). D. prolonged steroid therapy.
A. (1) (2) (4)
B. (2) (3) (4) The lamina dura is
C. (1) (2) (3) A. cortical bone.
D. All of the above. B. spongy bone.
C. immature bone.
D. a cribiform plate perforated by
nutrient canals.
E. None of the above.

[Type text]
Sinan Salim

A pleomorphic adenoma (mixed tumor) is A fracture through the angle of the mandible
most frequently found in the may result in an upward displacement of the
A. parotid salivary gland. proximal fragment. Which of the following
B. submandibular salivary gland. groups of muscles produce this movement?
C. sublingual salivary gland. A. Digastric and geniohyoid.
D. tongue. B. Masseter, digastric and lateral
(external) pterygoid.
Nicotinic stomatitis C. Masseter, temporal and lateral
A. is associated with squamous cell pterygoid.
carcinoma of the palate. D. Masseter, temporal and medial
B. is caused by smokeless tobacco. (internal) pterygoid.
C. appears clinically as small red dots E. None of the above.
surrounded by an elevated pale
mucosa. A unilateral fracture of the body of the
D. is characterized by dysplastic zygoma has the clinical sign of
changes. A. cerebrospinal rhinorrhea.
E. is irreversible. B. impaired hearing.
C. subconjunctival haemorrhage.
A radicular cyst D. otorrhea.
A. enlarges rapidly.
B. infiltrates bone. The most important clinical finding which
C. contains fluid. indicates the need for incision and drainage is
D. does not cause cortical bone A. trismus.
expansion. B. cellulitis.
E. is associated with a vital tooth. C. fluctuant swelling.
D. temperature above 37oC (98.6F).
Hypothyroidism affects dental development
by A 68-year old male develops a unilateral
A. causing microdontia. parotitis six days following an abdominal
B. delaying the eruption timetable. operation for intestinal obstruction. The
C. causing sclerotic bone to form over probable cause is
the occlusal surface of erupting teeth. A. an ascending infection of the parotid
D. accelerating the eruption timetable. duct from the mouth.
B. a suppurative parotitis secondary to
When given prophylactically to prevent bacteremia from the intestinal
infective endocarditis, oral penicillin should surgery.
be taken C. epidemic parotitis.
A. immediately prior to the dental D. calculus in parotid duct.
procedure. E. Mikulicz's disease.
B. one hour prior to the dental
procedure. During penicillin therapy, signs that may be
C. four hours prior to the dental considered as allergic manifestations are
procedure. A. deafness, dizziness and acute anemia.
D. one day prior to the dental procedure. B. crystalluria, nausea, vomiting and
E. two days prior to the dental anaphylactic shock.
procedure. C. oliguria, hematuria,
bronchoconstriction and
An examination of a patient who has fallen on cardiovascular collapse.
their chin and fractured the right subcondylar D. dermatitis, stomatitis,
region would reveal bronchoconstriction and
1. trismus. cardiovascular collapse
2. deflection of the mandible to the left on
opening. Which of the following is directly involved in
3. pain and tenderness. the conversion of prothrombin to thrombin?
4. paresthesia of the right lower lip. A. Sodium.
A. (1) (2) (3) B. Calcium.
B. (1) and (3) C. Fluoride.
C. (2) and (4) D. Potassium.
D. (4) only E. Bicarbonate.
E. All of the above.

[Type text]
Sinan Salim

In administering an inferior alveolar nerve Treatment of respiratory depression caused by


block, the needle penetrates the oral mucosa an overdose of barbiturates includes
and the A. coramine.
A. superior constrictor muscle lateral to B. meloxone.
the pterygomandibular raphe. C. oxygen.
B. buccinator muscle lateral to the D. caffeine.
pterygomandibular raphe.
C. pterygomandibular raphe. A well controlled insulin dependent diabetic
D. superior constrictor muscle medial to patient requires two teeth extracted under
the pterygomandibular raphe. local anesthesia at 9~a.m. Your instructions
E. internal pterygoid muscle. would include
A. eat breakfast, do not take insulin.
The lingual nerve contributes sensory fibers to B. take insulin, do not eat breakfast.
the C. eat breakfast and take insulin.
1. tongue. D. do not eat breakfast, do not take
2. lingual surface of the mandible. insulin.
3. floor of the mouth.
4. mandibular posterior teeth. The most common bacteria that develop
A. (1) (2) (3) resistance to antibiotics are
B. (1) and (3) A. streptococci.
C. (2) and (4) B. staphylococci.
D. (4) only C. lactobacilli.
E. All of the above. D. pneumococci.

Acetylsalicylic acid may be safely used as A patient under corticosteroid therapy requires an
an analgesic in patients who extraction under local anesthesia. You would
A. have a history of gastric ulcers. A. discontinue the corticosteroids a day before
B. are asthmatic. surgery.
C. have a bleeding disorder. B. reduce the dose to half the daily dose several
D. have emphysema (COPD). days before and after surgery.
E. have a duodenal ulcer. C. discontinue the corticosteroids and start the
patient on antibiotics right after surgery.
Which of the following is NOT associated D. proceed with the extraction without alteration of
with the administration of acetylsalicylic acid? the corticosteroid therapy.
A. Tinnitus.
B. Analgesia. A 10-year old boy complains of pain on the left
C. Antipyresis. side of his face. Examination reveals a tender
D. Methemoglobinemia. swelling which obliterates the angle of the
E. Inhibition of prostaglandin synthesis. mandible and elevates the lobe of the ear. Your
diagnosis is
Which of the following cysts is most likely to A. otitis media.
undergo transformation into an B. actinomycosis.
ameloblastoma? C. parotid neoplasm.
A. Radicular. D. epidemic parotitis.
B. Dentigerous.
C. Fissural. A patient presents with a diffuse cellulitis and
D. Traumatic lymphadenopathy. A superinfection with a
penicillinase-producing staphylococcus is
Which of the following penicillins is most suspected. Which of the following antibiotics
effective against gram-negative organisms? is the drug of choice for treating this
A. Nafcillin. condition?
B. Ampicillin. A. Tetracycline.
C. Methicillin. B. Cephalexin.
D. Penicillin V. C. Penicillin G.
E. Phenethicillin. D. Erythromycin.
E. Chloramphenicol

[Type text]
Sinan Salim

Which of the following has malignant potential? The occlusal surface of the second mandibular
A. Junctional naevus. primary molar when compared with that of its
B. Lentigo. permanent successor is
C. Periapical granuloma. A. larger mesiodistally.
D. White folded gingivo-stomatosis. B. equal mesiodistally.
C. narrower mesiodistally.
One millilitre of a 2% solution of lidocaine D. equal buccolingually and
hydrochloride contains how many milligrams mesiodistally.
of the drug?
A. 2 A child on antibiotic therapy would be more likely
B. 20 to develop which of the following diseases?
C. 200 A. Herpangina.
D. 2000 B. Pemphigus.
C. Moniliasis.
In effecting hemostasis, external cold D. Herpetic gingivostomatitis.
application produces
A. positive chemotaxis. When preparing a cavity in a deciduous molar, a
B. a transient vasoconstriction. dentist causes a small mechanical exposure of
C. increased vascular permeability. one of the pulp horns. There is a slight
D. accelerated healing. hemorrhage and the dentin surrounding the
exposure is sound. The treatment of choice is
Nitrous oxide A. extraction and space maintenance.
1. provides good muscle relaxation. B. pulp capping with calcium hydroxide and a
2. is non-explosive and non-flammable. base.
3. is a potent anesthetic. C. pulpectomy.
4. provides rapid induction and recovery. D. pulp capping with zinc oxyphosphate cement.
A. (1) (2) (3)
B. (2) and (3) An eight-year old patient has a coronal fracture,
C. (2) and (4) involving the enamel and dentin of the permanent
D. (4) only central incisor. When the acid etch technique is
E. All of the above. applied
A. a retentive pin is essential.
A nasopalatine nerve block will anesthetize the B. the dentin should be protected with zinc oxide
A. maxillary incisors and canines. and eugenol.
B. labial alveolar plate of the maxilla. C. the dentin should be protected with calcium
C. upper lip, nose and lower eyelid. hydroxide.
D. posterior portion of hard palate and D. the dentin does not need any covering.
overlying structures to the first
premolar. A 12-year old child presents at your office having
E. mucosa of the anterior palate. sustained a traumatic injury to his maxillary central
incisor. Clinically, the tooth is slightly mobile but has
In the pterygomandibular space, the inferior no coronal fracture. Radiographically there is a visible
alveolar nerve passes fracture of the root horizontally about 3 to 4 mm from
A. anterior to the deep tendon of the the tip. Electric pulp tests are inconclusive. The
temporal muscle. treatment of choice is to
B. lateral to the sphenomandibular A. do a vital pulpectomy and seal with gutta percha.
B. do a vital pulpectomy, seal with gutta percha and
ligament.
do an apical resection.
C. medial to the medial pterygoid
C. do a vital pulpectomy and seal with a silver point
muscle. to act as a splint.
D. medial to the pterygomandibular D. surgically remove the apical fracture only.
raphe. E. do nothing but keep the tooth under observation
radiographically.
During normal growth, the gnathion, as
viewed on successive cephalograms, will
move
A. downward and backward.
B. downward and forward.
C. backward and upward.
D. forward only.

[Type text]
Sinan Salim

The surface of a deciduous molar requiring the Which of the following can result in post
LEAST amount of reduction during the preparation developmental jaw growth?
of a stainless steel crown is the A. Hyperparathyroidism.
A. distal. B. Hyperthyroidism.
B. buccal. C. Adult hypothyroidism.
C. mesial. D. Osteitis deformans (Paget's).
D. lingual. E. Hypoparathyroidism.
E. occlusal.
Prior to the correction of a one-tooth anterior
Penicillin is most effective against cross-bite, the major factor to consider is the
A. fungi. A. adequacy of mesio-distal space.
B. viruses. B. developmental age of the patient.
C. gram-negative organisms. C. sequence of eruption of the
D. gram-positive organisms. permanent dentition.
D. morphology of the anterior teeth.
In a 4-year old child, the primary central incisor
has discoloured following a traumatic injury. The A Class I occlusion exists when the mesiobuccal
treatment of choice is cusp of the maxillary permanent first molar
A. pulpotomy. occludes
B. pulpectomy. A. distal to the buccal groove of the mandibular
C. observation. permanent first molar.
D. extraction. B. in the buccal groove of the mandibular
permanent first molar.
Primary teeth start to calcify in the fetus at C. mesial to the buccal groove of the mandibular
approximately the permanent first molar.
A. first month. D. lingual to the mesio-buccal cusp of the
B. third month. mandibular first molar.
C. fifth month.
D. seventh month. The most serious complication of an acute
periapical abscess of the maxilla is
Teeth generally in the process of calcification at A. cellulitis.
birth are all B. periostitis.
A. primary teeth. C. cavernous sinus thrombosis.
B. primary teeth and permanent first molars. D. spontaneous drainage of pus.
C. primary teeth and permanent central, lateral E. pericementitis.
and canine teeth.
D. primary teeth and all permanent teeth. A 9 year old patient presents with a submerged
non-carious deciduous first molar. The most
In females from age 6-12, the growth prediction of appropriate treatment is
mandibular intercuspid width will A. routine extraction and space maintenance.
A. decrease 1-2mm. B. surgical removal and space maintenance.
B. remain constant. C. extraction only.
C. increase 1mm. D. observation.
D. increase 2-3mm.
A corrected lingual anterior cross-bite of a
Mandibular growth maxillary central incisor is best retained by
A. is sustained over a longer period of time in girls. A. overcorrection.
B. is sustained over a longer period of time in B. an adequate anterior overbite.
boys. C. wearing a Hawley retainer.
C. occurs at the same chronologic age in both D. placement of a maxillary fixed
sexes. lingual arch.
D. occurs two years earlier in boys than in girls.

[Type text]
Sinan Salim

A Class II cavity prepared for an amalgam A ten-year old patient complains of discomfort in a
restoration in deciduous teeth does NOT require a maxillary deciduous second molar when eating.
gingival bevel because the enamel rods in the The tooth is mobile with a large mesio-occlusal
area incline amalgam restoration.
A. obliquely. You would suspect
B. gingivally. A. an exfoliating tooth.
C. horizontally. B. a hyperemic pulp.
D. vertically. C. a hyperplastic pulp.
D. an acute pulpitis.
A 10 year old has lost a mandibular deciduous first E. traumatic occlusion.
molar. The most appropriate treatment is
A. a stainless steel crown and loop Contraction of the external (lateral) pterygoid
space maintainer. muscle pulls the disc of the temporomandibular
B. an acrylic unilateral partial denture. joint
C. a band and loop space maintainer. A. forward and medially.
D. the removal of the corresponding tooth on the B. backward and medially.
other side of the arch. C. forward and laterally.
E. no treatment. D. backward and laterally.

The usual site for the development of bottle caries Which of the following drug groups is most likely to
syndrome is the cause xerostomia?
A. incisal edges of the mandibular incisors. A. Diuretics.
B. incisal edges of the maxillary incisors. B. Antibiotics.
C. gingival area of the mandibular incisors. C. Antidepressants.
D. gingival area of the maxillary incisors. D. Non-steroidal anti-inflammatory agents.

The anterior tooth with the greatest variability in In periodontal therapy, gingivectomy is the
size and shape is the treatment of choice to eliminate
A. maxillary lateral incisor. A. osseous craters.
B. mandibular canine. B. infrabony pockets.
C. maxillary canine. C. suprabony pockets.
D. maxillary central incisor. D. root hypersensitivity.
E. mandibular central incisor. E. inadequate attached gingiva.

Migration of the first permanent molar following the Salivary secretion


premature loss of the deciduous second molar is A. increases with age.
usually B. decreases with age.
A. mesial with the mesial buccal cusp rotating C. shows no difference with age.
lingually. D. shows a decrease in mineral content
B. mesial with the mesial buccal cusp rotating in the elderly.
buccally.
C. mesial with buccal tilt of the crown. On a periapical radiograph, the coronoid process
D. not found. may be superimposed over the apices of the
A. maxillary third molar.
The leeway space is defined as the space B. maxillary second premolar.
A. between the maxillary and mandibular occlusal C. mandibular third molar.
surfaces when the mandible is in rest position. D. mandibular first molar.
B. which results from the difference in sizes E. maxillary central incisors.
between the combined widths of the primary
cuspids and molars and the permanent cuspids The major vascular supply to the buccal gingiva is
and bicuspids. derived from
C. distal to the mandibular deciduous canine and A. intra-alveolar vessels.
mesial to the maxillary canine. B. superficial vessels over the periosteum.
D. between deciduous anterior teeth. C. vessels from the periapical area.
D. vessels of the periodontal ligament.

[Type text]
Sinan Salim

From the following, the EARLIEST clinical sign of In a teenage patient with normal gingivae,
gingivitis is interdental plaque removal is best accomplished
A. increased tooth mobility. with
B. change in color of the gingival margin. A. a regular toothbrush.
C. change in color of the attached gingiva. B. a hard nylon bristle brush.
D. change in consistency of the attached gingiva. C. a proxybrush.
D. floss.
Which of the following fibers make up the gingival E. interdental wood sticks.
collar (cuff)?
A. Circular. T lymphocytes play a primary role in
B. Dentogingival. A. antibody production.
C. Alveologingival. B. activation of complement.
D. Dentoperiosteal. C. immediate hypersensivity.
E. All of the above. D. antigen-antibody complexes.
E. lymphokine production and delayed
Chronic periodontitis is characterized as hypersensitivity.
A. inflammatory.
B. irreparable. The location and extent of sub-gingival calculus is
C. atrophic. most accurately determined clinically by
D. hyperplastic. A. radiopaque solution used in conjunction with
E. none of the above. radiographs.
B. disclosing solution.
In periodontal surgery, the most important clinical C. probing with a fine instrument.
factor is the relation of the base of the pocket to D. visual inspection.
the
A. calculus deposits. Gingivitis can be diagnosed radiographically by
B. enamel. A. loss of lamina dura.
C. cementum. B. horizontal bone resorption.
D. muco-gingival junction. C. thickening of the periodontal ligament space.
D. None of the above.
Periodontal pockets can be eliminated by
A. surgical resection of the pocket wall Clinical diagnosis of necrotizing ulcerative
(gingivectomy). gingivitis is confirmed by
B. apically positioned flap surgery. A. microbiological culture of organisms.
C. reattachment procedures. B. microscopic examination of a Gram stained
D. All of the above. smear.
E. None of the above. C. antibiotic sensitivity testing.
D. serological testing.
Before performing periodontal surgery, it is
important to A protective bite plate is indicated to
1. prescribe a mouthwash. A. reduce tooth mobility.
2. prescribe an oral antibiotic. B. prevent excessive tooth wear.
3. control plaque. C. control a bruxism habit.
4. scale and root plane. D. manage temporomandibular joint pain
A. (1) and (3) dysfunction syndrome.
B. (2) and (4) E. All of the above.
C. (3) and (4)
D. All of the above. CONTRAINDICATED for a 15-year old patient with
necrotizing ulcerative gingivitis who has no history
of previous gingival problems?
A. Antibiotic therapy.
B. Local debridement.
C. Topical steroid therapy.
D. Warm saline solution rinses.

[Type text]
Sinan Salim

Furcation involvement can be treated by In chronic periodontitis, the causative


1. root planing and oral hygiene instruction. organisms are found in
2. periodontal surgery with osseous and/or tooth A. the connective tissues of the gingiva.
recontouring. B. the periodontal ligament.
3. root resection. C. the alveolar bone.
4. antibiotic therapy only. D. the periodontal pocket.
A. (1) (2) (3) E. A. and D.
B. (1) and (3)
C. (2) and (4) In a removable partial denture, a palatal strap
D. (4) only is used instead of a narrow bar because it is
E. All of the above. A. more rigid with less apparent bulk.
B. easier to polish.
The periodontal ligament C. more stable.
A. achieves its final structural form just prior to D. less irritating to the soft tissues.
tooth eruption. E. more hygienic.
B. does not achieve its final structural form until
the tooth reaches a functional occlusion. Recurrence after surgery is a feature of the
C. does not contain lymphatic vessels. pleomorphic adenoma. The reason for this is
D. has a fast collagen turn-over time. the
A. tendency of the tumor to show hematogenous
Disuse atrophy of the periodontium causes metastases.
A. changes in the arrangement of fibre bundles. B. marked and early lymph node involvement.
B. narrowing of the periodontal ligament. C. anaplastic histologic characteristics of the
C. osteoporosis of the alveolar process. tumor.
D. decrease in tooth mobility. D. tendency to infiltrate the capsule of the gland.
E. All of the above.
Overextension of a mandibular denture base in the
Vitamin D deficiency in ADULTS causes distofacial area will cause dislodgement of the
A. acromegaly. denture during function as a result of the action of
B. myxedema. the
C. rickets. A. buccinator muscle.
D. osteomalacia. B. masseter muscle.
E. osteitis fibrosa cystica. C. pterygomandibular raphe.
D. lateral tendon of the temporal muscle.
Dental caries is associated with
1. certain strains of streptococci. The pontic of a fixed partial denture must
2. certain strains of staphylococci. A. touch the ridge with heavy pressure.
3. certain strains of lactobacilli. B. equal the buccolingual width of the natural
4. gram negative rods and certain anerobic tooth.
bacteria. C. be made entirely of cast metal.
A. (1) (2) (3) D. be no less than 3mm occlusogingivally.
B. (1) and (3) E. be designed to replace the lost tooth exactly.
C. (2) and (4)
D. (4) only Dental amalgam
E. All of the above. A. is almost insoluble in the oral fluids.
B. has a satisfactory compressive strength.
The epithelial attachment does not migrate apically C. adapts well to the walls of the prepared cavity.
in D. All of the above.
A. juvenile periodontitis.
B. hyperplastic gingivitis.
C. chronic periodontitis.
D. rapidly progressive periodontitis.

[Type text]
Sinan Salim

The smoothest surface finish on a composite resin Following orthodontic alignment, relapse of the
restoration will be produced by mandibular incisors cannot
A. a high speed multi-fluted tungsten A. be predicted from characteristics of the original
carbide bur. malocclusion.
B. a high speed fine diamond bur. B. occur if the second or third molars are removed.
C. pumice on rubber cup. C. occur if retainers are worn until the
D. fine aluminum-oxide disks. mandibular growth is complete.
E. a slow speed white stone. D. occur if a supracrestal fiberotomy is performed.

Before adhesion occurs between a liquid and a Aphthous stomatitis


solid, it is essential that the solid surface 1. is considered to be an autoimmune condition.
A. provides some mechanical interlocking with the 2. is more frequent in men than in women.
liquid. 3. may be related to the menstrual cycle.
B. exhibits a large contact angle with the liquid. 4. is rarely seen clinically with vesicle formation.
C. enters into some form of chemical reaction with 5. is of three to four days duration.
the liquid. A. (1) (2) (3)
D. be wetted by the liquid. B. (1) (3) (4)
C. (1) (3) (5)
Collagen D. (2) (3) (4)
A. is most common in hard tissues. E. (2) (3) (5)
B. forms insoluble high tensile strength fibres.
C. has a triple helical structure. Benzodiazepines have all of the following actions
D. All of the above. EXCEPT
A. muscle relaxation.
Which substance has the LEAST potential to B. sedation.
initiate dental caries? C. amnesia.
A. Lactose. D. anticonvulsant action.
B. Glucose. E. analgesia.
C. Maltose.
D. Xylitol. A patient who has until recently been on prolonged
corticosteroid therapy may have
Widening of the periodontal space is NOT seen A. increased bleeding time.
radiographically in B. hyposensitivity to pain.
A. trauma from occlusion. C. decreased tolerance to physiological stress.
B. orthodontic tooth movement. D. an increased metabolic rate.
C. scleroderma. E. high level of plasmatic cortisol.
D. Paget's disease.
Which of the following is a complication of
In chronic periodontitis, the predominant prolonged systemic corticosteroid treatment?
inflammatory cells in connective tissue are A. Oral candidiasis.
A. lymphocytes and plasma cells. B. Xerostomia.
B. neutrophils and macrophages. C. Aphthous stomatitis.
C. neutrophils and fibroblasts. D. Anorexia.
D. lymphocytes and histiocytes. E. Gingival hyperplasia.

In complete denture fabrication, insufficient space Which of the following is LEAST likely to influence
between the maxillary tuberosity and the the development of gingivitis?
retromolar pad would required A. Pregnancy.
A. opening the articulator. B. Diabetes.
B. avoiding covering the pad with the mandibular C. Traumatic occlusion.
base. D. Dental plaque.
C. not covering the tuberosity with the maxillary E. Calculus.
base.
D. surgically reducing the retromolar pad.
E. surgically reducing the maxillary tuberosity.

[Type text]
Sinan Salim

The maxilla is formed from This research study was


A. bundle bone. A. a retrospective study.
B. endochondral bone. B. a literature review.
C. membranous bone. C. an in-vitro study.
D. lamellar bone. D. a prospective study.

Which of the following has the strongest Koilonychia is


analgesic properties? A. bifid uvula.
A. Acetylsalicylic acid (325mg). B. auricular tags.
B. Ibuprofen (400mg). C. ankyloglossia.
C. Codeine (15mg). D. mulberry molars.
D. Acetaminophen (300mg). E. spoon-shaped (concave) nails.

After root planing and plaque control instruction, a What is the most important mechanical property to
22 year old patient still shows swollen and consider in the selection of an alloy for a long and
edematous gingiva with 3mm pockets and a 4 to narrow porcelain fused to metal bridge?
6mm band of attached gingiva. The most A. Elastic modulus.
appropriate treatment is B. Proportional limit.
A. an apically repositioned split C. Toughness.
thickness flap. D. Strength.
B. an apically repositioned full
thickness flap. Retentive pins
C. gingivectomy. A. assist in preventing shearing of nonrestored
D. further plaque control instruction. cusps.
E. occlusal adjustment. B. decrease the compressive strength of
the amalgam.
Healing following a gingivectomy occurs by C. increase the tensile strength of the
A. primary intention. amalgam.
B. secondary intention. D. provide resistance form to the
C. both primary and secondary preparation.
intention. E. decrease microleakage by bonding
D. tertiary intention. with the amalgam.

In a serial extraction procedure, after the maxillary A 30-year old HIV positive patient comes to your
first premolar has been extracted, the eruption office for the removal of an abscessed second
path of the maxillary canine will be molar. You should:
A. down and forward. A. Refer him to another dentist because your
B. down and backward. infection control procedures are inappropriate for
C. primarily forward. this type of condition.
D. primarily backward. B. Treat him at the end of the day.
C. Treat him in the same way you treat all your
An alveoplasty is performed to other patients.
1. facilitate removal of teeth. D. Double glove before starting any surgical
2. correct irregularities of alveolar ridges procedures.
following tooth removal.
3. prepare the residual ridge for dentures. Hypotension is associated with all of the
A. (1) and (2) following conditions EXCEPT
B. (1) and (3) A. anaphylactoid reaction.
C. (2) and (3) B. toxic reaction to local anesthetic.
D. All of the above. C. vasovagal shock (syncope).
D. hyperventilation.
Acanthosis is a thickening of the following layer
A. granular layer.
B. stratum corneum.
C. basal cell layer.
D. prickle cell layer.

[Type text]
Sinan Salim

While removing an impression from the oral cavity, The most common initial sign of occlusal trauma is
some material falls in the throat. The patient A. tooth mobility.
coughs and breathes. You should B. tooth sensitivity.
A. push once on the abdomen at the level of the C. radiographic evidence of increased
umbilicus. periodontal space.
B. push 6 to 10 times on the abdomen at the level D. loss of pulp vitality.
of the diaphragm.
C. slap the patient in the back between the During an injection at the mandibular foramen, the
shoulder blades. needle passes through the buccal mucosa and the
D. make the patient drink cold water buccinator muscle and is
without breathing. A. external to the pterygomandibular raphe and
E. instruct the patient to continue coughing. external to the medial (internal) pterygoid muscle.
B. external to the pterygomandibular raphe and
An overhang may result even though a wedge is internal to the medial (internal) pterygoid muscle.
correctly used for a gingivally extended MO C. internal to the pterygomandibular raphe and
restoration on tooth 2.4 because internal to the medial (internal) pterygoid muscle.
A. spherical alloy is used. D. internal to the medial (internal) pterygoid
B. the root proximity of the canine muscle and lateral to the neck of the condyle.
makes using a wedge difficult.
C. there is often an open contact between the A 13 year old male complains of red, bleeding and
canine and first premolar. swollen gums. Clinical examination reveals this is
D. there is a concavity on the mesial root surface present only on the labial gingiva of the maxillary
of the premolar. anterior teeth. What is the most likely etiologic
factor?
Acute osteomyelitis of the mandible differs from A. Blood dyscrasia.
malignant neoplasm because it B. Insulin dependent diabetes mellitus.
A. is asymptomatic. C. Mouth breathing habit.
B. is associated with high fever. D. Pubertal hormones.
C. has an excellent prognosis.
D. has well defined radiographic margins. The most appropriate treatment for an
endodontically treated molar with an existing MOD
Osteosarcoma differs from fibrous dysplasia restoration is a
because it A. cast gold inlay.
A. can invade soft tissue. B. bonded composite resin.
B. has ill-defined radiographic margins. C. bonded amalgam.
C. can be radiolucent, mixed or radiopaque. D. cast restoration with cusp coverage.
D. is difficult to curette from normal bone.
In administering an inferior alveolar nerve block,
According to the Canadian Dental Associations trismus may occur if the needle penetrates the
Code of Ethics, a dentist can A. superior constrictor muscle lateral to the
A. split the fees with referring dentists. pterygomandibular raphe.
B. collect the third party payment without B. buccinator muscle lateral to the
requesting the patient's portion. pterygomandibular raphe.
C. bill an insurance company a higher than usual C. superior constrictor muscle medial to the
fee, if it benefits the patient. pterygomandibular raphe.
D. charge different fees when warranted by clinical D. buccinator muscle mesdial to the
conditions. pterygomandibular raphe.
E. internal pterygoid muscle.
Which of the following is NOT suggestive of
a diagnosis of acute necrotizing ulcerative What is the most common site for intraoral
gingivitis (ANUG)? squamous cell carcinoma?
A. Bleeding from the gingiva. A. Gingiva.
B. "Punched-out" papillae with necrotic slough. B. Floor of mouth.
C. Fetor oris. C. Buccal mucosa.
D. Metallic taste. D. Dorsum of tongue
E. Periodontal pocketing.

[Type text]
Sinan Salim

Condensing osteitis (focal sclerosing Which impression material can be stored for more
osteomyelitis) differs from idiopathic than 24 hours, be poured and still produce
osteosclerosis because it is accurate dies?
A. expansile. A. Polysulfide.
B. painful. B. Condensation reaction silicone.
C. associated with fever. C. Reversible hydrocolloid.
D. associated with pulpal inflammation. D. Addition reaction silicone (polyvinylsiloxane).
E. Irreversible hydrocolloid.
A patient complains of intermittent spontaneous
pain in a tooth that was previously treated with a Relapse of rotations of teeth corrected by fixed
direct pulp cap. The tooth is not sensitive to orthodontic treatment is best minimized by
percussion. Both hot and cold produce severe A. removal of third molars and occlusal
pain. The most likely diagnosis is equilibration.
A. reversible pulpitis. B. wearing retainers indefinitely with
B. irreversible pulpitis. circumferential supra-crestal fiberotomy of rotated
C. cracked tooth syndrome. teeth.
D. acute apical periodontitis. C. overcorrection during active treatment.
D. circumferential supra-crestal fiberotomy of all
Conversion of a flush terminal plane to a teeth.
Class I occlusion without orthodontic intervention
is a result of A 32 year old female patient complains of fever,
A. a closure of mandibular primate spaces. weight loss and general malaise. She has a rash
B. mandibular forward growth greater on the malar area and nose, as well as some
than maxillary forward growth. irregularly shaped ulcerations on the buccal
C. mesial movement of the mandibular secondary mucosa. The most likely diagnosis is
first molars following exfoliation of the mandibular A. lichen planus.
second primary molars. B. lupus erythematosus.
D. distal movement of the secondary maxillary first C. erythema multiforme.
molars as the maxillary premolars erupt. D. bullous pemphigoid.
E. pemphigus.
The most compelling diagnostic signs and/or
symptoms indicative of a pterygomandibular space A patient complains of irritability, fatigue and
infection is/are weakness. She is losing weight and has diarrhea.
A. trismus without swelling. The clinical examination shows diffuse brown
B. trismus with swelling in the macular pigmentation of the oral mucosa. The
submandibular region. pigmentation appeared recently. The most likely
C. swelling in the submandibular region without diagnosis is
trismus. A. iron deficiency anemia.
D. swelling in the infraorbital region without B. Addisons disease.
trismus. C. acute myeloid leukemia.
D. Crohns disease.
A mother is concerned because her 6 year old child
has a double row of teeth. On examination, the child A 27 year old woman complains of burning mouth,
has a complete primary dentition with teeth 4.1 and fatigue, palpitations and lack of energy. The
3.1 erupting lingual to 8.1 and 7.1. Teeth 8.1 and 7.1 clinical examination shows angular cheilitis and
demonstrate significant mobility and radiographically atrophic glossitis. What is the most likely
have one quarter of the root remaining. The incisor diagnosis?
segment is not crowded. The most appropriate A. Iron deficiency anemia.
management for this patient is to B. Crohns disease.
A. reassure the parent that this occurrence is a
C. Chronic lymphocytic leukemia.
variation of normal eruption.
D. Plummer-Vinson syndrome.
B. disc teeth 8.2 and 7.2 to allow eruption of 4.1 and
3.1.
C. extract teeth 8.1 and 7.1 at this appointment. A 32 year old male complains of weight loss and
D. refer the patient to an orthodontist as soon as diarrhea. The clinical examination shows
possible. lymphadenopathy, multiple flat erythematous lesions
on the palate and a linear gingival
erythema. What is the most likely diagnosis?
A. Crohns disease.
B. Diabetes mellitus.
C. AIDS.
D. Leukemia.

[Type text]
Sinan Salim

Which of the following compounds released by Phenytoin (Dilantin) treatment causes gingival
inflammatory cells induces bone resorption? hyperplasia as a result of
A. Nitric oxide. A. increased number of epithelial cells.
B. Interleukin - 1. B. increased production of collagen.
C. Bradykinin. C. thickening of the basement membrane.
D. Alkaline phosphatase. D. increased production of proteoglycans.

The pain associated with pulpitis often disappears While setting, a gypsum-bonded investment
when a patient visits a dental office. This occurs material will undergo an additional expansion if it
due to which of the following events? occurs
A. Stress-induced sympathetic activity inhibits A. under water.
pulpal sensory fibres that cause pain. B. under vacuum.
B. Stress-induced sympathetic activity causes C. in a cold environment.
vasodilatation of pulpal blood vessels. D. in a dry environment.
C. Local mediators of pulpal pain are not released
when a stress-induced rise in blood pressure During the extraction of an impacted tooth 3.8 the
occurs. lingual nerve is damaged. All of the following can
D. Pulpal pain receptors undergo fatigue. occur EXCEPT a
A. loss of taste from the anterior 2/3 on the left
A pale 8 year old patient presents with generalized side of the tongue.
gingival enlargement and spontaneous bleeding. B. deviation of the tongue to the left on protrusion.
The most appropriate initial management of this C. decreased salivary output from the left
patient is to sublingual gland.
A. perform an incisional biopsy. D. decreased salivary output from the left
B. obtain a cytologic smear. submandibular gland.
C. order a complete blood count with a differential. E. numbness of the floor of the mouth on the left
D. obtain bacterial cultures. side.
E. order fasting blood glucose levels.
The most important factor in stainless steel crown
A patient has a displaced right subcondylar retention in a primary tooth is the
fracture of the mandible. On opening, the mandible A. preservation of the coronal bulge.
deflects to the right. Which muscle is prevented B. maintenance of parallelism.
from functioning appropriately? C. placement of accessory grooves.
A. Medial pterygoid. D. insertion of retentive pins.
B. Temporalis.
C. Masseter. A 3 year old, 16kg child is given 2 cartridges of 4%
D. Lateral pterygoid. prilocaine with 1:200,000 epinephrine for
extraction of deciduous teeth. After a few minutes
A 20-year old female student presents with clinical he becomes lethargic, disorientated and eventually
symptoms of acute necrotizing ulcerative gingivitis begins to convulse. The most probable cause of
(ANUG). Her food intake for the last 24 hours this reaction is
indicates a soft diet lacking in fruits and A. epinephrine.
vegetables. The patients diet is important to B. allergic reaction to the local anesthetic.
investigate further because C. overdose of local anesthetic.
A. a deficiency of certain nutrients causes ANUG. D. epilepsy precipitated by epinephrine
B. ANUG may be limiting the foodchoices the
patient is making. Generalized malaise and elevated body
C. ANUG can be cured through modification of temperature may be associated with
diet. A. periapical granuloma.
D. patients with ANUG lose interest in eating. B. acute dento alveolar abscess.
C. acute suppurative pulpitis.
D. chronic ulcerative pulpitis.

[Type text]
Sinan Salim

Which of the following statements concerning the The epithelial cells which proliferate in apical
airway is correct? granulomas originate from
A. Sympathetic fibres constrict the bronchioles. A. surface epithelium.
B. The trachea is membranous posteriorly to B. odontogenic epithelial rests.
accommodate the pulsations of the aorta. C. odontoblasts.
C. The right primary bronchus forms two D. mesenchymal cells.
secondary bronchi.
D. An aspirated foreign body would likely fall into A well-defined unilocular radiolucency in the area
the right primary bronchus of a congenitally missing tooth, when all adjacent
teeth are vital, is most likely a/an
Which primary molar terminal plane relationship is A. residual cyst.
most likely to develop into an B. odontogenic keratocyst.
Angle Class II molar relationship? C. dentigerous cyst.
A. Straight. D. radicular cyst.
B. Distal step. E. periapical granuloma.
C. Mesial step.
D. Marked mesial step. Leukoplakia is best managed by
A. observation.
Xerostomia potentially can contribute to B. replacement of amalgam restorations.
1. increased caries. C. changes in diet.
2. altered taste. D. incisional biopsy.
3. increased candidal infections. E. carbon dioxide ablation.
4. burning mouth.
A. (1) (2) (3) Chronic periodontitis is characterized as
B. (1) and (3) A. inflammatory.
C. (2) and (4) B. irreparable.
D. (4) only C. atrophic.
E. All of the above. D. hyperplastic.

An angina attack in a patient with a known cardiac Amalgam


problem immediately after a tooth extraction was 1. spherical alloys shrink slightly when setting.
most probably precipitated by 2. lathe cut alloys expand slightly when setting.
A. a sudden repositioning of the patient into an 3. admix alloys are dimensionally stable when
upright position. setting.
B. a delayed allergic reaction to the anesthetic 4. high copper content alloys have improved
agent. resistance to tarnish and corrosion.
C. the stress resulting from the A. (1) (2) (3)
treatment. B. (1) and (3)
D. the use of a vasoconstrictor in the anesthetic C. (2) and (4)
agent. D. (4) only
E. All of the above.
Incorporation of alumina to dental ceramics is used
to improve Which of the following should be performed to
A. translucency. confirm a diagnosis of pseudomembranous
B. strength. candidiasis?
C. adhesive potential. A. A biopsy submitted for viral testing.
D. marginal fit. B. A cytological smear stained for fungal
microorganisms.
Thumb sucking, if persistent and aggressive, tends C. An excisional biopsy submitted for microscopic
to create examination.
A. skeletal open bite. D. A swab submitted for bacterial culture.
B. posterior crossbite.
C. proclined mandibular anterior teeth.
D. retroclined maxillary anterior teeth.

[Type text]
Sinan Salim

A patient presents with a chief complaint of Where is periapical cemental dysplasia most
severe pain in my right ear which began when commonly found?
eating, three hours ago. An examination reveals A. Maxillary anterior region.
tenderness over the right preauricular region, B. Mandibular anterior region.
maximum interincisal opening of 21mm with C. Mandibular premolar region.
deflection to the right, right lateral excursion of D. Maxillary posterior region
9mm and left lateral excursion of 2mm. The most
likely diagnosis is Tetracycline
A. left anterior disc displacement with reduction. A. is bactericidal.
B. right anterior disc displacement with reduction. B. disrupts the bacterial cell wall.
C. left anterior disc displacement without C. interferes with bacterial protein synthesis.
reduction. D. can be given in conjunction with penicillin.
D. right anterior disc displacement without
reduction. Overcontouring in the gingival third of a provisional
restoration contributes to the
A ghost-like opaque image in a panoramic A. strength of the material of the gingival margin.
radiograph caused by a metal earring worn in the B. accumulation of plaque and gingival
lobe of the left ear will be superimposed over the inflammation.
A. left mandibular ramus. C. displacement of the gingiva for future
B. right mandibular ramus. impression making.
C. left posterior maxilla. D. marginal integrity of the provisional restoration.
D. right posterior maxilla.
Osteoporosis is linked to
During radiographic film processing, silver halide is A. low estrogen levels.
removed from the emulsion during the B. high testosterone levels.
A. developing stage. C. low androgen levels.
B. post-developing wash stage. D. high progesterone levels.
C. fixing stage.
D. post-fixing wash stage. Nonsteroidal anti-inflammatory drugs
(NSAIDS) act by inhibiting the synthesis of
Ground glass is the classical description of the A. phospholipase A2.
radiographic appearance found in B. arachidonic acid.
A. acute osteomyelitis. C. cyclooxygenase.
B. fibrous dysplasia. D. prostaglandins.
C. early periapical cemento-osseous dysplasia. E. leukotrienes.
D. Pagets disease of bone.
In comparing dimensions of the primary
The coronoid process of the mandible can be seen mandibular second molar to its permanent
in the following types of radiographs EXCEPT successor, the primary molar has a/an
A. periapical. A. greater crown length occlusoapically.
B. maxillary occlusal. B. equal root length occlusoapically.
C. panoramic. C. greater crown width mesiodistally.
D. lateral cephalometric D. equal crown width buccolingually.

In root planing, the working edge of the blade of a All afferent impulses from the pulp result in
Gracey curette creates the following angle with the the sensation of
root surface: A. heat.
A. 30 50 degrees. B. pain.
B. 60 80 degrees. C. proprioception.
C. 90 100 degrees. D. cold.
D. 110 120 degrees.

[Type text]
Sinan Salim

The most appropriate emergency management of A patient with a history of shortness of breath and
a mature permanent tooth with acute irreversible ankle edema is probably suffering from
pulpitis is A. asthma.
A. pulpotomy. B. emphysema.
B. pulpectomy. C. rhinophyma.
C. incision and drainage. D. cardiac insufficiency
D. trephination.
E. apical surgery. Osseous resective surgery is best suited for
periodontal sites with
When removing teeth from a mandible which has A. severe attachment loss.
been exposed to ionizing radiation therapy, the B. deep intrabony defects.
most serious complication is C. teeth with short roots.
A. fracture. D. early to moderate bone loss.
B. osteomyelitis.
C. prolonged hemorrhage. The predominant immunoglobulin isolated from
D. localized osteitis. saliva is
A. IgG.
The most important factor to consider before B. IgM.
extracting a mandibular incisor is C. IgA.
A. severity of the crowding. D. IgD.
B. mandibular curve of Spee.
C. a Boltan discrepancy. The biologic width on average is
D. the vertical incisor relationship. A. 1mm.
E. the horizontal incisor relationship. B. 2mm.
C. 3mm.
A single adjustment of an expansion screw in a D. 4mm.
removable appliance for dental arch expansion will
cause an expansion of "Dens in dente" (dens invaginatus) is most
A. 0.25mm. commonly associated with
B. 0.5mm. A. supernumerary teeth.
C. 0.75mm. B. dentinogenesis imperfecta.
D. 1mm. C. osteogenesis imperfecta.
D. anterior teeth.
A patient who has a retrognathic facial type and a E. amelogenesis imperfecta.
convex profile has a
A. prominent nose. Polymerization shrinkage in a composite resin is
B. deficient midface. reduced by
C. horizontal growth pattern. A. placing a glass ionomer liner on all exposed
D. retruded mandible. dentin before placing composite resin.
B. doubling the curing time of the resin in
If removal of teeth is indicated in a patient who is preparations that are deep.
to receive radiation therapy for a carcinoma of the C. using a flowable composite on the gingival floor
tongue, the teeth should be extracted of Class II preparations.
A. prior to the radiation therapy. D. incremental placement of no more than 2mm
B. during the radiation therapy. thickness of composite resin.
C. immediately post radiation therapy.
D. six months post radiation therapy. The risk of contracting hepatitis B through a
needlestick injury from a chronic hepatitis B carrier
What is the most important mechanical property to is increased when the patients serology report
consider in the selection of an alloy for a long and indicates the presence of
narrow porcelain fused to metal bridge? A. HBsAg.
A. Elastic modulus. B. anti-HBcAg.
B. Proportional limit. C. HBeAg.
C. Toughness. D. anti-HBsAg.
D. Tensile strength. strength only in other Q

[Type text]
Sinan Salim

Local anesthetics During a surgical procedure, a severed artery is


A. do not readily pass the blood-brain barrier. best treated by
B. interfere with the propagation of action A. pressure over adjacent bone.
potentials in nerve fibres. B. a pressure pack.
C. selectively interfere with the propagation of C. clamping with a hemostat.
action potentials in nociceptive fibres. D. clamping and ligation
D. do not have an effect on any other tissue than
the nervous tissue. Which of the following has the LEAST potential for
soft tissue damage?
Salicylates are preferred over acetaminophen for A. 35% hydrogen peroxide.
their B. 37% phosphoric acid.
A. antiinflammatory action. C. 9% hydrofluoric acid.
B. antipyretic action. D. 20% aluminium chloride.
C. antiemetic action.
D. antitussive action. What is the most appropriate location for the
placement of a retentive pin?
The management of syncope following local A. At the dentino-enamel junction.
anesthetic administration does NOT B. In dentin a minimum 0.5mm from the dentino-
include enamel junction.
A. elevating the legs. C. In enamel a minimum 0.5mm from the dentino-
B. placing in a supine position. enamel junction.
C. administering oxygen. D. At least 2.5mm from the cavosurface margin.
D. ensuring the airway is open.
E. administering epinephrine What is the name of the process by which
carbamide peroxide bleaches teeth?
The therapeutic International Normalized A. Oxidation.
Ratio (INR) for atrial fibrillation is B. Addition.
A. less than 2.0. C. Subtraction.
B. 2.0 to 3.0. D. Hydrogenation.
C. 3.0 to 4.0.
D. 4.0 to 5.0. A 67 year old patient with xerostomia presents with
root caries. The most appropriate material to
In an acute periradicular abscess, which of the restore these lesions is
following teeth is most likely to spread infection to A. flammable composite resin.
the submandibular space? B. hybrid composite resin.
A. Mandibular second bicuspid. C. silver amalgam.
B. Maxillary third molar. D. glass ionomer cement.
C. Mandibular first molar.
D. Mandibular third molar. A Class II amalgam restoration in primary molars
E. Mandibular lateral incisor. should include
A. divergent buccal and lingual walls in the gingival
The most common site for breast carcinoma to to occlusal direction.
metastasize to the maxillofacial regions is B. an axial wall that follows the dentino-enamel
A. anterior maxilla. junction.
B. anterior mandible. C. an isthmus that occupies 2/3 of the
C. posterior maxilla. intercuspal distance.
D. posterior mandible. D. undercut enamel and dentin.
E. gingiva.
Which of the following is associated with folate
Proliferative periostitis (Garr's osteomyelitis) deficiency?
A. is associated with neoplasia. A. Birth defects.
B. occurs in the debilitated elderly patient. B. Microcytic anemia.
C. involves cortical bone and periosteum. C. Low serum homocysteine.
D. produces a pathologic fracture of bone. D. Elevated high density lipoproteincholesterol.
E. is associated with fever.

[Type text]
Sinan Salim

Vitamin D is mainly activated in the During a single appointment, a dentist restores a


A. skin upon ultraviolet radiation from the sun. maxillary incisor with two separate Class IV
B. liver upon hydroxylation. composite resin restorations. Which of the
C. pancreas upon hydroxylation. following procedures should be submitted to the
D. intestinal mucosa upon absorption. patients dental insurance provider?
A. Two three surface restorations.
A patient seeks advice from a dentist about the B. Two four surface restorations.
alternate sweetener, sucralose. Which of the C. One five surface restoration.
following statements is correct about sucralose? D. One six surface restoration.
A. It breaks down at high temperatures and cannot E. One eight surface restoration.
be used in baking.
B. It is a derivative of sucrose. A patient with a displaced fracture of the left
C. It is twice as sweet as table sugar. condylar neck of the mandible is expected to
D. It causes diarrhea when ingested in excessive A. deviate to the right on protrusion.
amounts. B. deviate to the left on opening.
C. have normal right lateral excursion.
Selective grinding for equilibrating complete D. have premature occlusal contact(s) on the right
dentures is most accurate when the dentures are posterior.
A. rearticulated using original jaw relation records.
B. rearticulated with a new series of jaw relation Which of the following can cause a crenated
records. (scalloped) tongue?
C. equilibrated in the patient's mouth. A. Erythema migrans.
D. equilibrated after the patient has worn the B. Fissured tongue.
dentures several days. C. Median rhomboid glossitis.
D. Macroglossia.
According to the Canadian Dental
Associations Code of Ethics, a dentist can Which of the following antibiotics may be cross-
A. split the fees with referring dentists. allergenic with penicillin?
B. collect the third party payment without A. Neomycin.
requesting the patient's portion. B. Cephalexin.
C. bill an insurance company a higher than usual C. Clindamycin.
fee, if it benefits the patient. D. Erythromycin.
D. charge different fees when warranted by clinical E. Tetracycline.
conditions.
Which of the following anatomic structures is/are
On a bite-wing radiograph, a smooth surface potentially affected by administration of local
proximal carious lesion in enamel appears as a anesthesia?
triangle with the A. Internal maxillary artery.
A. base at the dentino-enamel junction. B. Pterygoid plexus of veins.
B. base facing toward the pulp. C. Posterior superior alveolar nerve.
C. apex pointing to the enamel surface. D. All of the above.
D. apex pointing to the dentino-enamel
Junction Which of the following is suggestive of a sialolith in
the submandibular duct?
With the exception of third molars, of the ages A. Periodic swelling of the cheek.
listed below, what is the earliest age when the B. Fluctuant bluish swelling in the floor of the
crowns of all permanent teeth are completely mouth.
calcified? C. Drainage of pus from Stensen's duct.
A. 4 to 5 years. D. Pain and swelling associated with eating.
B. 6 to 7 years.
C. 8 to 9 years. Nitrous oxide, when used as a sedative,
D. 11 to 12 years. produces
A. euphoria.
B. dizziness.
C. lethargy.
D. anesthesia.

[Type text]
Sinan Salim

Which one of the following agents does NOT After the crown completion stage, trauma to a
cause gastric irritation? developing tooth may be responsible for
A. Aspirin. A. enamel hypoplasia.
B. Alcohol. B. gemination.
C. Ibuprofen. C. dilaceration.
D. Indomethacin. D. fusion.
E. Acetaminophen.
Secondary dentin will develop
Dentigerous (follicular) cysts should be completely A. if the teeth become abraded.
enucleated because B. if cavities develop.
A. the epithelium of the cyst can degenerate and C. following fractures.
form toxic substances. D. due to chemical irritation.
B. the epithelial lining of the cyst has the potential E. All of the above.
for neoplastic change.
C. the connective tissue of the cyst wall can In the young adult dentition, an obtuse gonial
become osteoblastic. angle is associated with a tendency towards
D. continued growth is likely to result in a A. mandibular prognathism.
supernumerary tooth. B. deep overbite.
C. normal occlusion.
Fractures of the mandible passing through tooth D. Class II malocclusion.
sockets fall into the classification of
A. simple. The anterior component of force may be observed
B. compound. clinically as
C. greenstick. A. distal movement of a permanent
D. comminuted. mandibular cuspid.
E. displaced. B. mesial movement of a permanent
maxillary first molar.
Post-extraction alveolar osteitis or "dry socket" is C. A. and B.
clinically characterized by D. None of the above.
A. an indurated swelling and fever.
B. intermittent, sharp, lancinating pain with no Following the normal loss of the mandibular
swelling. second deciduous molar, the mandibular first
C. ecchymosis of the oral mucous membranes permanent molar
with local tenderness. A. remains fixed in its place.
D. continuous severe pain with no swelling. B. migrates mesially.
E. a socket oozing blood-tinged fluid and pus. C. migrates distally to make room for the
permanent premolar.
Death from barbiturates is the result of D. migrates the same distance as the maxillary
A. alkalosis. first permanent molar.
B. irreversible hypotension.
C. toxic effects on the liver. A unilateral fracture of the mandibular condyle in a
D. depression of the centres of respiration. child
E. allergy. A. will not affect future mandibular growth.
B. will result in ankylosis of the joint.
Conscious sedation differs from general C. may result in asymmetrical mandibular growth.
anesthesia in that the D. inhibits mandibular molar development.
A. patient retains all reflexes. E. will result in a Class II malocclusion.
B. patient is not responsive.
C. patient's pain threshold is not altered. Which of the following fibers make up the gingival
D. patient's heart rate is increased. collar (cuff)?
A. Circular.
In festooning and trimming a stainless steel crown, B. Dentogingival.
special attention must be paid to the greater length C. Alveologingival.
necessary in the region of the mesiobuccal bulge D. Dentoperiosteal.
in the primary E. All of the above.
A. mandibular first molar.
B. mandibular second molar.
C. maxillary canine.
D. maxillary lateral incisor.

[Type text]
Sinan Salim

In periodontal surgery, the most important clinical Which of the following would differentiate clinically
factor is the relation of the base of the pocket to between an acute periapical abscess and an acute
the periodontal abscess on a single rooted tooth?
A. calculus deposits. A. Palpation.
B. enamel. B. Use of a local anesthetic.
C. cementum. C. Percussion.
D. muco-gingival junction. D. Pulp vitality testing.

The primary factor for selecting periodontal flap narrow slit-like areas of hingival recession over
surgery rather than gingivectomy is the roots of teeth are called
A. presence of gingival edema. A. festoons
B. pocket depth. B. clefts.
C. presence of subgingival calculus. C. craters.
D. need for access to bony lesion. D. fenestrations.
E. dehiscences.
The immediate treatment of a periodontal abscess
is to The treatment of choice of a deep pocket whose
A. provide drainage. base extends beyond the attached gingiva is
B. prescribe an analgesic. A. gingivectomy.
C. relieve the occlusion. B. mucogingival surgery.
D. prescribe an antibiotic. C. root planing.
E. prescribe an antibiotic and relieve the D. None of the above.
occlusion.
Trauma from occlusion may be diagnosed
The functions of the periodontal ligament include radiographically by evidence of
1. regeneration. A. cemental tears.
2. proprioception. B. horizontal bone loss.
3. support. C. widening of the periodontal ligament space.
4. taste perception. D. narrowing of the periodontal ligament space
A. (1) (2) (3)
B. (1) and (3) Gingivitis is a reversible form of periodontal
C. (2) and (4) disease. Gingivitis does not necessarily progress
D. All of the above. to periodontitis.
A. The first statement is true, the second false.
Plaque becomes more cariogenic when B. The first statement is false, the second true.
A. it has become heavily calcified. C. Both statements are true.
B. pyogenic organisms predominate. D. Both statements are false
C. acidogenic bacteria and fermentable
carbohydrate are present.
D. it is heavy and associated with an orange stain.

Fluorides are effective in the prevention of dental


caries by
A. increasing the resistance of dentin to bacterial
penetration.
B. causing tooth enamel to be more resistant to
demineralization.
C. providing a more favorable pulpal blood supply.
D. All of the above.

In synthesizing and secreting antibodies, the most


active cell is the
A. mast cell.
B. macrophage.
C. eosinophilic granulocyte.
D. plasma cell.
E. T-cell lymphocyte.

[Type text]

Vous aimerez peut-être aussi